Você está na página 1de 52

TH

APGO MEDICAL STUDENT EDUCATIONAL OBJECTIVES, 10 EDITION


TEACHING CASES
!

U N I T 4: R E P R O D U C T I V E E N D O C R I N O L O G Y , I N F E R T I L I T Y &
RELATED TOPICS

Educational Topic 45: Normal and


Abnormal Uterine Bleeding

Rationale: The occurrence of bleeding at times other than expected menses is common. Accurate diagnosis of
abnormal uterine bleeding is necessary for appropriate management.

Intended Learning Outcomes:!!

A student should be able to:

• Define the normal menstrual cycle and describe its endocrinology and physiology
• Define abnormal uterine bleeding
• Describe the pathophysiology and identify etiologies of abnormal uterine bleeding
• Describe the steps in the evaluation and initial management of abnormal uterine bleeding
• Summarize medical and surgical management options for patients with abnormal uterine bleeding

TEACHING CASE

CASE: A 45 year-old G2P0020 woman, with LMP 21 days ago, presents with heavy menstrual bleeding. Prior to 6
months ago her cycles occurred every 28-30 days, lasted for 6 days, and were associated with cramps that were relieved
by Ibuprofen. In the last 6 months there has been a change with menses occurring every 25-32 days, lasting 7-10 days
and associated with cramps not relieved by ibuprofen, passing clots and using two boxes of maxi pads each cycle. She is
worried about losing her job if the bleeding is not better controlled. She denies dizziness, but complains of feeling weak
and fatigued. Her weight has not changed in the last year. She denies any bleeding disorders or reproductive cancers in
the family. She uses condoms for contraception. She takes no daily medications and has no other medical problems.
She is married and works in a factory.

On physical exam, her weight is 150 pounds, height is 5 feet, 6 inches, BP 130/88, P 100. She appears pale. Pelvic exam
shows normal vulva, vagina and cervix; normal sized, non-tender, mobile uterus; non-tender adnexae without palpable
masses.
TH
APGO MEDICAL STUDENT EDUCATIONAL OBJECTIVES, 10 EDITION
TEACHING CASES

COMPETENCY-BASED DISCUSSION & KEY TEACHING POINTS:

Competencies addressed:
• Patient Care
• Medical Knowledge
• Systems-based Practice

1. What are the parameters of a normal menstrual cycle?

• Interval 21-35 days (Mean: 28 days)


• Duration: 2-7 days (Mean: 5 days)
• Volume: <80ml (Mean 35 ml)
• Composition: Non-clotting blood, endometrial debris

2. Describe the normal endocrinologic and physiologic events that make the menstrual cycle possible.

• The menstrual cycle can be divided into two portions. From the perspective of the endometrium, the cy-
cle consists of the proliferative phase and the secretory phase. From the perspective of the ovary, the cycle
is composed of the follicular phase and the luteal phase. The two phases are demarcated by ovulation.
Thus, the proliferative phase corresponds to the follicular phase and the secretory phase corresponds to
the luteal phase.
• Day 1 is the first day of bleeding. In a 28 day cycle, ovulation occurs on Day 14. During the early follicu-
lar phase, increasing FSH drive the growth of a cohort of follicles. The increase in follicles drives a corre-
sponding increase in estradiol. As estradiol increases, the endometrium proliferates and hypertrophies in
response. FSH decreases in response to the negative inhibitory effect of estradiol. As a result the follicle,
which is most sensitive to FSH, becomes dominant, continuing to secrete estradiol. This is the follicle
destined for ovulation. The massive amount of estradiol causes the LH surge which signals ovulation or
the release of the oocyte.
• The corpus luteum is formed at the ovulation site and produces progesterone. This progesterone trans-
forms the endometrium to make it receptive to implantation. If pregnancy does not occur, then the cor-
pus luteum undergoes atresia with a consequent fall in progesterone. This progesterone withdrawal
causes the endometrium to shed. The fall in progesterone also allows FSH to rise and a new cohort of fol-
licles to develop, and a new cycle begins.

3. What is the definition of abnormal uterine bleeding?

• Menstrual bleeding which falls outside the normal parameters is considered abnormal. Menorrhagia is
prolonged excessive bleeding. Metrorrhagia is irregular or intermenstrual bleeding. The combination of
these is menometrorrhagia. Bleeding that occurs after menopause has occurred is also considered abnor-
mal uterine bleeding.

4. What possible etiologies could cause this patient’s bleeding?

PALM-COEIN is an acronym that was published in 2011 by the International Federation of Gynecology and
Obstetrics that was created for the purpose of establishing a universally accepted nomenclature to describe
uterine bleeding abnormalities.

Copyright © 2014 by Association of Professors of Gynecology and Obstetrics (APGO).


For permissions: apgoadmin@apgo.org!
TH
APGO MEDICAL STUDENT EDUCATIONAL OBJECTIVES, 10 EDITION
TEACHING CASES

• PALM-Structure Causes
! Polyp
! Adenomyosis
! Leiomyoma
! Malignancy and Hyperplasia
• COEIN-Nonstructural Causes
! Coagulopathy
! Ovulatory Dysfunction
! Endometrial
! Iatrogenic
! Not Yet Classified

5. Which are the potential etiologies of ovulatory dysfunction?

• Hyperandrogenic anovulation (polycystic ovary syndrome, congenital adrenal hyperplasia, or androgen-


producing tumors)
• Hypothalamic dysfunction (i.e. due to anorexia nervosa)
• Hyperprolactinemia
• Thyroid disorder
• Primary pituitary disease
• Premature ovarian failure
• Iatrogenic (due to radiation or chemotherapy)
• Medications

6. Discuss the mechanism for anovulatory bleeding

• Progesterone withdrawal signals the endometrium to shed in a uniform way by causing spiral artery
spasm. Women who do not ovulate do not experience progesterone withdrawal because they do not
form a corpus luteum and usually have bleeding due to unopposed estrogen with either estrogen with-
drawal or estrogen excess. Neither of these mechanisms causes spiral artery spasm, and therefore can re-
sult in non-uniform shedding of the lining at irregular intervals.

7. How can you tell if this patient is having ovulatory cycles?

• History consistent with ovulatory cycles (regular, presence of molimina)


• Timed (luteal phase) endometrial biopsy- is it secretory?
• LH surge kits (ovulation prediction kits) detect LH surge in urine which follows LH surge in serum but
occurs before ovulation
• Basal body temperature chart with small temperature increase (0.5 degrees) after ovulation
• Day 21 serum progesterone level

8. What are the appropriate lab tests that should be ordered in this patient?

• CBC, TSH, Prolactin


• Pregnancy Test
• Endometrial Biopsy
• Pelvic Ultrasound

Copyright © 2014 by Association of Professors of Gynecology and Obstetrics (APGO).


For permissions: apgoadmin@apgo.org!
TH
APGO MEDICAL STUDENT EDUCATIONAL OBJECTIVES, 10 EDITION
TEACHING CASES

9. What further tests would you order based on the following results?

Labs show Hgb: 9.0, HCT: 27%, HCG: negative, TSH and Prolactin are within normal limits. Endometrial bi-
opsy shows normal secretory endometrium, Pelvic ultrasound shows a normal sized uterus with a heterogene-
ous myometrium, the endometrial lining is 1.4 cm and irregular consistent with endometrial polyp, normal
ovaries.

• Fluid-enhanced sonohysterogram
• Hysterosalpingogram
• Diagnostic hysteroscopy

10. Describe potential treatment options for this patient.

Certain etiologies will respond better to certain therapies. Ablation is most effective when there is no anatom-
ic lesion. In this patient’s case, because she likely has an anatomic abnormality, one may consider offering a
hysteroscopy or a hysterectomy (if she does not desire childbearing and desires definitive treatment). Medical
options include the following:

• Oral contraceptive pills


• Cyclic progestin
• GnRH agonist
• High dose NSAID’s
• Tranexamic acid
• Levonorgestrol IUD (Mirena)

However, since the etiology of her abnormal uterine bleeding is likely an endometrial polyp, medical manage-
ment is really only an option as temporizing measures if she is not a surgical candidate.

11. What are important considerations when counseling the patient and helping her choose the best option for her?

• Fertility: The patient’s desire for future childbearing should be assessed


• Therapeutic goals: The patient should consider how permanent a solution she desires. The various pos-
sible therapies are associated with a failure rate and a recurrence risk.
• Operative risks: Patients who have significant comorbidities or who are severely anemic should approach
surgical therapies carefully.
• Time to menopause: The length of time until likely menopause should be discussed with patient and
should be taken into consideration in the patient who might be hesitant to pursue surgical therapy.

REFERENCES

Beckman CRB, et al. Obstetrics and Gynecology. 7th ed. Philadelphia: Lippincott, Williams & Wilkins, 2013.

Hacker NF, Moore JG, et al. Essentials of Obstetrics and Gynecology. 5th ed. Philadelphia: Saunders, 2010.

ACOG Practice Bulletin 136, Management of Anovulatory Uterine Bleeding Associated with Ovulatory Dysfunction,
July 2013.

ACOG Practice Bulletin 128, Diagnosis of Abnormal Uterine Bleeding in Reproductive-Aged Women, 2012.

Copyright © 2014 by Association of Professors of Gynecology and Obstetrics (APGO).


For permissions: apgoadmin@apgo.org!
TH
APGO MEDICAL STUDENT EDUCATIONAL OBJECTIVES, 10 EDITION
TEACHING CASES
!

U N I T 3: G Y N E C O L O G Y
SECTION B: BREASTS

Educational Topic 40:


Disorders of the Breast

Rationale: Breast disorders and concerns are common. They are often distressing and may indicate the presence of
serious disease.

Intended Learning Outcomes:!!

A student should be able to:

• List factors that place individuals at risk for breast disorders


• Describe symptoms and physical examination finding of benign or malignant conditions of the breast
• Demonstrate the performance of a clinical breast examination
• Discuss the steps in the evaluation of common breast complaints: mastalgia, mass, nipple discharge
• Discuss initial management options for benign and malignant conditions of the breast

TEACHING CASE

CASE: A 56-year-old woman G0P0 woman made an appointment to see her gynecologist because of a small lump in
her right breast that she has been able to feel for 2 months. She has not had prior breast problems and does not have a
family history of breast cancer. There are no apparent skin changes, asymmetry or skin dimpling. Axillary or supracla-
vicular lymph nodes are not palpable. Breasts are symmetric, diffusely cystic and non-tender. There is a firm area ap-
proximately 1 cm in diameter with indiscreet borders at the 9 o’clock position on her right breast. The area is slightly
different in consistency than the rest of the surrounding tissue. The patient’s mammogram revealed dense breast tissue,
but no mammographic abnormalities.

COMPETENCY-BASED DISCUSSION & KEY TEACHING POINTS:


Competencies addressed:
• Patient Care
• Medical Knowledge
• Interpersonal and Communication Skills
• Professionalism
• Systems-based Practice
TH
APGO MEDICAL STUDENT EDUCATIONAL OBJECTIVES, 10 EDITION
TEACHING CASES

1. What is the proper technique to perform a breast exam?

• Inspection (with patient sitting): examination of contour, symmetry, skin changes or nipple retraction
• Palpation (in upright and supine positions) of breast, areola and nipples
• Examination of axillae and supraclavicular fossae

2. What are common risk factors for breast cancer?

• The commonly quoted risk of breast cancer (1 in 8 women) represents a cumulative lifetime risk.
• For a woman aged 50-59 years, the lifetime risk of having a breast cancer diagnosis is 1 in 36,
while for a woman-aged 70-79 years, the risk increases to 1 in 24.
• Risk factors include
• Patient age
• Smoking
• Prior personal history
• Mammographic breast density
• Family history
• Genetics (BRCA 1 and 2 mutations)
• Nulliparity
• Late childbearing (first pregnancy >30 years)
• Early menarche
• Late menopause
• Fibrocystic changes with atypia
• History of breast radiation
• Hormone exposure
• Obesity
• Excessive alcohol use (> 2 drinks/day)

3. What is your next step in this patient’s management?

• A persistent palpable breast mass requires evaluation. Mammography alone is not sufficient to rule out
malignancy in a patient with a palpable breast mass.
• Ultrasonography, magnified mammographic imaging or MRI of the breast may provide addi-
tional information.
• Further, these studies may identify cystic structures or variations in breast architecture that ac-
count for the palpable abnormality.
• When cyst aspiration is performed, the fluid may be discarded if it is clear (not bloody) and the
mass disappears following the aspiration. Otherwise, the patient should have a breast biopsy.
• Solid masses require histologic diagnosis in most cases. Fine-needle aspiration or stereotactic
needle biopsy may be an alternative to open breast biopsy.
• If breast cancer or specific benign condition is not detected by fine-needle aspiration or needle
core biopsy, open biopsy is necessary.

4. What are some common benign breast disorders?

• Fibrocystic changes: most common benign breast conditions; commonly present as cyclic, bilateral, pain
and engorgement
• Non-proliferative lesion (RR of developing breast ca 1.0)

Copyright © 2014 by Association of Professors of Gynecology and Obstetrics (APGO).


For permissions: apgoadmin@apgo.org!
TH
APGO MEDICAL STUDENT EDUCATIONAL OBJECTIVES, 10 EDITION
TEACHING CASES

• Proliferative lesion (hyperplasia) without atypia (RR 1.5-2.0)


• Atypical hyperplasia (RR 8-10)
• Fibroadenoma: second most common benign breast condition
• Fibrous and glandular tissue
• Occurs in young women
• Firm, painless, mobile mass
• Intraductal Papilloma
• Commonly found in peri and menopausal patients
• Bloody, serous or turbid nipple discharge
• Excisional biopsy is often needed
• Galactocele
• Cystic dilatation of a duct filled with thick, milky fluid
• Secondary infection causes mastitis
• Needle aspiration is often curative

REFERENCES

Beckman CRB, et al. Obstetrics and Gynecology. 7th ed. Philadelphia: Lippincott, Williams & Wilkins, 2013.

Hacker NF, Moore JG, et al. Essentials of Obstetrics and Gynecology. 5th ed. Philadelphia: Saunders, 2010.

CDC Breast Cancer Screening http://www.cdc.gov/cancer/breast/basic_info/screening.htm

CME breast health modules http://www.medscape.com/editorial/public/breastcancer-cdc

Copyright © 2014 by Association of Professors of Gynecology and Obstetrics (APGO).


For permissions: apgoadmin@apgo.org!
TH
APGO MEDICAL STUDENT EDUCATIONAL OBJECTIVES, 10 EDITION
TEACHING CASES
!

UNIT%5:%NEOPLASIA%

Educational Topic 52:


Cervical Disease and Neoplasia

Rationale: Early recognition and proper evaluation of pre-invasive cervical disease and cancer can reduce morbidity
and mortality.

Intended Learning Outcomes:!!

A student should be able to:

• Describe the pathogenesis of cervical cancer


• Identify the risk factors for cervical neoplasia and cancer
• List the guidelines for cervical cancer screening
• Describe the initial management of a patient with an abnormal Pap test
• Describe the symptoms and physical findings of a patient with cervical cancer

TEACHING CASE

CASE: A generally healthy 26 year-old G1P0 woman with a last menstrual period approximately 16 weeks ago is re-
ferred for the management of an abnormal Pap test showing High Grade Squamous Intraepithelial Lesion (HGSIL).
This Pap test was obtained 10 weeks ago when she underwent an elective termination of an unplanned pregnancy at
approximately six weeks of gestation. She has not had any prior Pap tests. She has never been tested for sexually
transmitted infections. The combination of the undesired pregnancy and the abnormal Pap test, however, has been a
“wake-up call” and today she requests testing for “everything.” She received Depo-Provera at the time of the termina-
tion, and has not had a period yet. She reports a history of normal, regular menses and has used oral contraceptives
inconsistently in the past. She began having sexual intercourse at the age of 17, and has had 4 lifetime partners. She is
on no other medications and has no known drug allergies. Her family history is notable for a grandmother with breast
cancer. She smokes ½ pack of cigarettes per day, does clerical work for a moving company, and is engaged to be mar-
ried in 6 months.
TH
APGO MEDICAL STUDENT EDUCATIONAL OBJECTIVES, 10 EDITION
TEACHING CASES

COMPETENCY-BASED DISCUSSION & KEY TEACHING POINTS:


Competencies addressed:
• Patient Care
• Medical Knowledge
• Systems-based practice

1. According to recent guidelines published by the American College of Obstetricians and Gynecologists (2012), how
many Pap tests should this patient have had given her age and clinical history?

• First cytology should be obtained at age 21 regardless of coitarche.


• Between the ages of 21 and 29, there is no benefit to annual screening; screening with cytology alone eve-
ry 3 years is recommended. It leads to harm due to overtreatment of screen detected abnormalities.
• Women ages 30–65 years should be screened with cytology and HPV testing (“cotesting”) every 5 years
(preferred) or cytology alone every 3 years.
• Women over 65 years of age with evidence of adequate negative prior screening and no history of CIN2+
within the last 20 years should not be screened for cervical cancer with any modality. Once screening is
discontinued it should not resume for any reason, even if a woman reports having a new sexual partner.
• This patient should have had only two screening pap tests by now.

2. Which historical risk factors does this patient have for having cervical dysplasia or for having cervical dysplasia
progress to cervical cancer?

• She has poor compliance with screening, early age of coitarche (< 19 years of age), and is a cigarette smok-
er
• Abnormal Pap test is presumptive evidence of HPV infection
• She is at risk of other sexually transmitted infections given her lack of barrier contraception, including
HIV/AIDS
• Number of lifetime sexual partners
• Low socio-economic status and poor access to healthcare

3. What are other possible risk factors for development of cervical dysplasia?

• She probably does not have an autoimmune disease, given her generally healthy medical history. Other
diagnoses that would increase her risk of cervical neoplasia include SLE, and history of organ transplanta-
tion on immunosuppressive therapies.
• DES exposure
• HIV infection

4. What is meant by the term "high-grade squamous intraepithelial lesion"?

• Each Pap test report should have a statement of specimen adequacy (satisfactory, unsatisfactory), general
categorization (negative for intraepithelial lesion or malignancy, epithelial cell abnormality, other), and
interpretation/result (negative for intraepithelial lesion or malignancy, epithelial cell abnormalities). Pos-
sible Pap test results include: ASCUS, ASC-H, LGSIL, HGSIL, AGC, AIS, and squamous cell carcinoma.
• Each category of abnormal cytologic reading encompasses a spectrum of possible correlating pathologic
(histologic) diagnosis that should be further explored and identified. In this case, the finding of HGSIL
encompasses moderate and severe dysplasia, carcinoma in situ (CIN 2 and CIN 3).

Copyright © 2014 by Association of Professors of Gynecology and Obstetrics (APGO).


For permissions: apgoadmin@apgo.org!
TH
APGO MEDICAL STUDENT EDUCATIONAL OBJECTIVES, 10 EDITION
TEACHING CASES

• Cells were identified on cytology (Pap test) suggesting abnormal cellular maturation between 1/3 and full
thickness of the squamous epithelial layer of the cervix.

5. What would you recommend as the next step in the evaluation of this patient's abnormal Pap test?

• Abnormal Pap test results require further work-up, typically to establish a diagnosis. This patient will re-
quire colposcopy and directed biopsies, including an endocervical curettage (ECC). Once a diagnosis is
made based on these findings, appropriate treatment can then be recommended.
• Available algorithms for abnormal cytologic and pathologic cervical neoplasia are detailed from ASCCP
(see references).
• Patient should also be counseled about STI testing (including HIV), smoking cessation, and use of barrier
contraception.

6. Would typing for the human papilloma virus (HPV) aid in the management of this patient?

• HPV testing should not be used to screen women between the ages of 21-29, either as a stand-alone test
or as a cotest with cytology. In this patient with HSIL, there is no role for HPV testing, as the result is ex-
pected to be positive. This patient requires colposcopic examination. For LSIL, HPV can be expected to
be positive in 77% of cases, making this test impractical in deciding to triage to colposcopy.
• Low risk HPV types include 6 and 11, are associated with cervical warts. High risk HPV types include 16
and 18, are associated with high grade cervical dysplasia and cervical cancer.

REFERENCES

Beckman CRB, et al. Obstetrics and Gynecology. 7th ed. Philadelphia: Lippincott, Williams & Wilkins, 2013.

Hacker NF, Moore JG, et al. Essentials of Obstetrics and Gynecology. 5th ed. Philadelphia: Saunders, 2010.

ACOG Practice Bulletin 131, Screening for Cervical Cancer, November, 2012.

Saslow D., et al. American Cancer Society, American Society for Colposcopy and Cervical Pathology, and American
Society for Clinical Pathology Screening Guidelines for the Prevention and Early Detection of Cervical Cancer. Journal
of Lower Genital Tract Disease 2012;16(30:175-204.

Copyright © 2014 by Association of Professors of Gynecology and Obstetrics (APGO).


For permissions: apgoadmin@apgo.org!
TH
APGO MEDICAL STUDENT EDUCATIONAL OBJECTIVES, 10 EDITION
TEACHING CASES
!

U N I T 2: O B S T E T R I C S
SECTION B: ABNORMAL OBSTETRICS

Educational Topic 15: Ectopic Pregnancy

Rationale: Ectopic pregnancy is a leading cause of maternal morbidity and mortality. Early diagnosis and management
may prevent serious adverse outcomes, and may preserve future fertility.

Intended Learning Outcomes:!!

A student should be able to:

• Develop a differential diagnosis for vaginal bleeding and abdominal pain in the first trimester
• Perform a physical exam to assess for acute abdomen
• List risk factors for ectopic pregnancy
• Discuss diagnostic protocols for ectopic pregnancy
• Describe treatment options for patients with ectopic pregnancy

TEACHING CASE

CASE: A 36-year-old G1P0010 woman presents to the office with onset of light vaginal bleeding, which she feels is not
her menstrual period, and mild right lower quadrant pain, which she rates as 2/10. The pain is intermittent and
crampy, and is not associated with urination. There is no nausea or vomiting. The patient’s last bowel movement was
yesterday and was normal in consistency without blood or black color.

Note that the LNMP was not given in the clinical case to see if the students ask about this. You can tell
them that the LNMP was 7 weeks ago.

Her past medical history is notable for no allergies, no medications, and two hospitalizations. The first was eight years
ago for lower abdominal pain which was thought to be due to pelvic inflammatory disease and which resolved with
antibiotics. The second was for a left ectopic pregnancy that required surgical removal of her left tube.

Review of systems and family history are unremarkable. Social history reveals that she is mutually monogamous with a
male partner without contraception.

Physical examination shows an anxious appearing female with a temperature of 99.2 ° F, orally, a BP of 105/62, and a
pulse of 95. Examination of her abdomen reveals normal bowel sounds. There are no masses, organomegaly, disten-
tion, or rebound tenderness. She has mild discomfort in the right lower quadrant. Pelvic examination reveals right
TH
APGO MEDICAL STUDENT EDUCATIONAL OBJECTIVES, 10 EDITION
TEACHING CASES

adnexal tenderness without adnexal masses. Uterus is of normal size and there is discomfort on cervical motion. The
rectal exam is negative with heme negative stool.

COMPETENCY-BASED DISCUSSION & KEY TEACHING POINTS:


Competencies addressed:
• Patient Care
• Medical Knowledge
• Systems-Based Practice

1. What is the differential diagnosis for this patient? What aspects of her history and physical examination might lead
you to be suspicious of an ectopic pregnancy? (*indicates signs or symptoms for teaching case)
• Differential Diagnosis:
• Obstetrical:
! Ectopic pregnancy
! Incomplete, completed, or missed abortion
! Threatened abortion
• Gynecological:
! Ovarian cyst
! Adnexal torsion
! Pelvic inflammatory disease
! Endometriosis
• Other Non Ob/Gyn:
! Appendicitis
! Inflammatory bowel disease
! Urinary tract infection
! Bladder stone
• Symptoms or clinical presentation may include:
• Abdominal pain (95-100%)*
• Abnormal uterine bleeding (65-85%)*
• Amenorrhea (75-95%)
• Signs or physical findings include:
• Abdominal tenderness* (80-90%)
• Adnexal tenderness* (75-90%)
• Normal uterine size* (70%)
• Adnexal mass (30-50%)

Ectopic pregnancies often have an atypical presentation and can have extensive overlap with other abdominal and
pelvic disorders.

Key Teaching Point: Any sexually active woman in the reproductive age group who presents with pain, ir-
regular bleeding, and/or amenorrhea should have ectopic pregnancy as part of the initial differential diagno-
sis. Pain is common but is not always the presenting symptom in ectopic pregnancies.

Copyright © 2014 by Association of Professors of Gynecology and Obstetrics (APGO).


For permissions: apgoadmin@apgo.org!
TH
APGO MEDICAL STUDENT EDUCATIONAL OBJECTIVES, 10 EDITION
TEACHING CASES

2. What are the risk factors for ectopic pregnancy and which of these risk factors does the patient have (* indicates
patient risk factors for teaching case)?

• Previous ectopic pregnancy (approx 10 times increase)*


• History of pelvic inflammatory disease, gonorrhea, or chlamydia infections*
• History of previous gyn or abdominal surgery*
• Sterilization failure
• Endometriosis
• Congenital uterine malformation
• Assisted reproductive technology
• Older age (35-44 y/o are 3 times higher risk than younger women)*

Key Learning Point: Diagnosing and managing a patient with an ectopic pregnancy can be difficult.
Knowledge of the risk factors can heighten the clinician’s suspicion and may determine the direction of the
investigation. Ectopic pregnancy is the leading cause of pregnancy-related death in the first trimester.

3. Where can ectopic pregnancies occur and how frequently does this happen?
• Ampullary, 80%
• Isthmic, 12%
• Fimbrial, 5%
• Cornual/Interstitial, 2%
• Abdominal, 1.4%
• Ovarian, 0.2%
• Cervical, 0.2%

Key Learning Point: Ectopic pregnancies do not have to be confined to the fallopian tube.

4. What initial test would you order for this patient to assist you in narrowing down your diagnosis?

• Quantitative β-hCG (in order to rule in or rule out an intrauterine pregnancy with transvaginal ultra-
sound, the β-hCG needs to be greater than 1500 mIU/ml)

Key Learning Point: Confirming pregnancy is critical in the diagnosis of ectopic pregnancy. If this test is not
ordered in a timely manner it can lead to significant morbidity and mortality.

5. If this patient’s test is positive, what tests could be helpful in making a more definitive diagnosis?

• STAT CBC (to check for anemia that may indicate intra-abdominal bleeding)
• Transvaginal ultrasound to look for intrauterine pregnancy or extrauterine pregnancy (assuming that the
quantitative β-hCG > 1500mIU/ml an ectopic pregnancy can be diagnosed if there is no evidence of an
intrauterine pregnancy on transvaginal ultrasound)
• Serial Quantitative β-hCG levels: If the level is equivocal and the ultrasound is not helpful, monitoring
the β-hCG level rise in 48 hours can aid in distinguishing between a viable intrauterine pregnancy and
non-viable intrauterine pregnancy or ectopic pregnancy. In viable early intra-uterine pregnancy, hCG
levels will usually rise by at least 66% in 48 hours. A β-hCG level less than 66% should cause suspicion of
ectopic or non-viable intrauterine pregnancy. Patients who are stable where the diagnosis is unclear can
be followed by serial β-hCG levels and, when levels have reached high enough for ultrasound to be effec-
tive, can have repeat ultrasounds
• Serum progesterone level may be helpful in some situations.
Copyright © 2014 by Association of Professors of Gynecology and Obstetrics (APGO).
For permissions: apgoadmin@apgo.org!
TH
APGO MEDICAL STUDENT EDUCATIONAL OBJECTIVES, 10 EDITION
TEACHING CASES

Key Learning Point: If a diagnosis of ectopic pregnancy is suspected, it is critical to ensure that the patient is
stable. The next step in confirming the diagnosis is to perform the necessary tests. Ultrasound and serial β-
hCG are essential tests in determining whether a pregnancy is intra-uterine or extra-uterine when the diag-
nosis is uncertain.

6. What options are available for the management of ectopic pregnancy?

• Check for Rh status and give Rh negative women Rho-GAM to prevent isoimmunization
• Medical treatment: methotrexate
• Hemodynamically stable patient
• Quantitative β-hCG (higher failure rate if β-hCG is greater than 5,000 mIU/ml thus multiple doses
may be required)
• No fetal heart beat seen outside of the uterus
• Ectopic gestation that is not too big (usually <3.5cm).
• Cooperative patient who will be sure to return for appropriate follow up and blood work
• Surgical treatment: Laparoscopy or Laparotomy with or without conservation of the fallopian tube
• Expectant management is an option if β-hCG is low and decreasing and patient is willing to take the risk
of tubal rupture and hemorrhage.

Key Learning Point: Management of ectopic pregnancy is determined by the patient’s history and physical
examination as well as ultrasound and laboratory investigations.

REFERENCES

Beckman CRB, et al. Obstetrics and Gynecology. 7th ed. Philadelphia: Lippincott, Williams & Wilkins, 2013.

Hacker NF, Moore JG, et al. Essentials of Obstetrics and Gynecology. 5th ed. Philadelphia: Saunders, 2010.

ACOG Practice Bulletin 94, Medical Management of Ectopic Pregnancy, June 2008 (reaffirmed 2012).

e-Medicine. Sepilian V, Wood E. Ectopic pregnancy. May 7, 2010.

Copyright © 2014 by Association of Professors of Gynecology and Obstetrics (APGO).


For permissions: apgoadmin@apgo.org!
TH
APGO MEDICAL STUDENT EDUCATIONAL OBJECTIVES, 10 EDITION
TEACHING CASES
!

U N I T 3: G Y N E C O L O G Y
SECTION A: GENERAL GYNECOLOGY

Educational Topic 38:


Endometriosis

Rationale: Endometriosis may result in pelvic pain, infertility and menstrual dysfunction.

Intended Learning Outcomes:!!

A student should be able to:

• Describe theories of the pathogenesis of endometriosis.


• List the most common sites of endometriosis
• Describe the symptoms and physical examination findings in a patient with endometriosis.
• Describe the diagnosis and management options of endometriosis.

TEACHING CASE

CASE: A 28-year-old woman G0P0 woman is seen because of the inability to conceive for the past two years. She has
never used oral contraceptives and she and her husband have not used any form of birth control for over two years. Her
menarche occurred at the age of 12 and her menses became very painful in her late teens. She has had chronic cyclical
pelvic pain, which has progressively worsened over the years. This pain is incapacitating at times. She describes the
location of the pain to be in the lower abdomen and pelvis that radiates into the lower back. In addition to the pain, her
menstrual periods have become increasingly frequent and heavy. She experiences deep dyspareunia that began with her
first sexual partner and has continued with her husband. She denies any non-cyclical vaginal bleeding, discharge and
weight loss. She states that her 22-year-old younger sister has always had very painful menses.

On physical examination the patient looks her age. She is 138 lbs and is 5’6”. Her BP is 110/76 mm Hg with a heart rate
of 85 bpm. Her head and neck examination is negative. Cardiac and respiratory systems are also normal. Examination
of the abdomen reveals that it is flat with no scars. On palpation she has generalized tenderness of the lower abdomen.
There are no signs suggesting evidence of a surgical abdomen and she has no costo-vertebral angle tenderness. The
pelvic exam showed a fixed, retroverted uterus. The uterosacral ligaments on both sides are nodular. A 5 cm right ad-
nexal mass is palpated and tender.

Transvaginal ultrasound of the pelvis showed a 5.5 cm cystic mass with low-level echoes in the right ovary. The left
ovary was reported as normal. The uterus is retroverted and is of normal size and contour. There is no evidence of
fibroids and the endometrial lining is normal.
TH
APGO MEDICAL STUDENT EDUCATIONAL OBJECTIVES, 10 EDITION
TEACHING CASES

COMPETENCY-BASED DISCUSSION & KEY TEACHING POINTS:


Competencies addressed:
• Patient Care
• Medical Knowledge
• Systems-Based Practice

1. What symptoms does this patient present with that would lead to a suspicion of endometriosis?

• Infertility
• Dysmenorrhea
• Cyclic lower abdominal and pelvic pain
• Back pain
• Dyspareunia
• Possible family history

Key Learning Point: Endometriosis can manifest in many ways. Patients may have no symptoms to signifi-
cant symptoms and sequelae.

2. Describe the physical findings for this patient that helps confirm a possible diagnosis of endometriosis?

• Tender nodular uterosacral ligaments on pelvic and rectovaginal exam


• Fixed, retroflexed uterus
• Ovarian mass

Key Learning Point: Endometriosis can manifest in many ways. Patients may present from the range of no
physical findings to fixed locked non-mobile pelvic structures. The most common sites of endometriosis are
the ovaries, uterine ligaments, and recto and vesicovaginal septae.

3. After discussing the possibility of endometriosis, the patient asks, “How did I get this disease?” How do you answer
the patient?

• Attachment and implantation of endometrial glands and stroma to peritoneal tissue from retrograde men-
strual flow
• Hematogenous and lymphatic spread
• Stem cells from bone marrow
• Coelomic metaplasia

Key Learning Point: Retrograde menstrual flow is the most likely pathogenesis of endometriosis.

4. What alternative diagnoses would you consider in this patient?

• Chronic pelvic pain-consider chronic pelvic inflammatory disease, adhesions, gastrointestinal conditions,
interstitial cystitis.
• Dysmenorrhea-consider causes of primary and secondary dysmenorrhea
• Dyspareunia-consider chronic pelvic inflammatory disease, or ovarian cysts.

Key Learning Point: Many conditions both gynecologic and non-gynecologic can present similar to endome-
triosis with the reverse also being true.

5. How is the diagnosis of endometriosis made?


Copyright © 2014 by Association of Professors of Gynecology and Obstetrics (APGO).
For permissions: apgoadmin@apgo.org!
TH
APGO MEDICAL STUDENT EDUCATIONAL OBJECTIVES, 10 EDITION
TEACHING CASES

• History and physical exam are first steps due to variety of presentations
• Direct visualization is needed for establishing a diagnosis
• Tissue biopsy makes definitive diagnosis
• Pelvic sonogram cannot make diagnosis, but can exclude other conditions

Key Learning Point: Endometriosis is a tissue diagnosis that requires a biopsy. A presumptive diagnosis is
made based on history and physical examination.

6. What protocols are used to stage endometriosis?

• The revised American Fertility Society’s (AFS) staging system is generally used to stage endometriosis in
the infertile patient. In the AFS system, points are assigned for size and depth of implants and for the se-
verity of adhesions in various locations. Stages I through IV are assigned on the basis of points. Manage-
ment of endometriosis can be guided by the stage of disease and the desire for fertility.
• The American Society of Reproductive Medicine (ASRM) protocol correlates fertility potential with a
quantified stage of disease. The staging includes the color of lesions, the percentage of surface involved
and a detailed description of endometriomas.

Key Learning Point: Systematic protocols are used in staging the severity of endometriosis.

7. What are the treatment options for a patient with a diagnosis of endometriosis?

• Depends on presenting symptoms and severity, location and severity, desire for future childbearing, age,
and possible gastrointestinal or urinary tract involvement
• Expectant management may be considered for patients with minimal symptoms and disease and/or pa-
tients who are trying to conceive
• Medical therapy includes NSAID’s, combined estrogen and progestin contraceptives, progestins alone,
danazol (17-alpha ethinyl testosterone derivative), and gonadotropin-releasing hormone (GnRH)
• Surgical management ranges from conservative options such as removal of endometriomas and destruc-
tion of endometriotic implants to definitive approaches such as hysterectomy with bilateral salpingoopho-
rectomy

Key Learning Point: The management of endometriosis is dependent on a number of factors including
wish for fertility preservation, symptom severity, and the location of endometriosis.

REFERENCES

Beckman CRB, et al. Obstetrics and Gynecology. 7th ed. Philadelphia: Lippincott, Williams & Wilkins, 2013.

Hacker NF, Moore JG, et al. Essentials of Obstetrics and Gynecology. 5th ed. Philadelphia: Saunders, 2010.

ACOG Practice Bulletin 114, Management of Endometriosis, July 2010.

Copyright © 2014 by Association of Professors of Gynecology and Obstetrics (APGO).


For permissions: apgoadmin@apgo.org!
TH
APGO MEDICAL STUDENT EDUCATIONAL OBJECTIVES, 10 EDITION
TEACHING CASES
!

U N I T 4: R E P R O D U C T I V E E N D O C R I N O L O G Y , I N F E R T I L I T Y &
RELATED TOPICS

Educational Topic 48: Infertility

Rationale: The evaluation and management of an infertile couple requires an understanding of the processes of
conception and embryogenesis, as well as sensitivity to the emotional stress that can result from the inability to
conceive.

Intended Learning Outcomes:!!

A student should be able to:

• Define infertility
• List the causes of male and female infertility
• Describe the evaluation and initial management of an infertile couple
• Describe the psychosocial issues associated with infertility
• Describe management options for infertility
• Describe ethical issues confronted by patients with infertility
• Identify the impact of genetic screening and testing on infertility associated treatments.

TEACHING CASE

CASE: A 37‐year-old woman and her 37‐year‐old male partner present with the complaint of a possible fertility prob-
lem. The couple has been married for 2 years. The patient has a 4-year-old daughter from a previous relationship. The
patient used birth control pills until one‐and‐a- half years ago. The couple has been trying to conceive since then and
report a high degree of stress related to their lack of success. The patient reports good health and no problems in con-
ceiving her previous pregnancy or in the vaginal delivery of her daughter who has cystic fibrosis. She reports that her
periods were regular on the birth control pill, but have been irregular since she discontinued taking them. She reports
having periods every 5‐7 weeks. She works as a cashier, runs 12-24 miles each week for the last 2 years, and has no his-
tory of STIs, abnormal Paps, smoking, alcohol or other drugs. She has had no surgery. She has been taking a multi-
vitamin with folic acid since trying to conceive.

The patient’s partner also reports good health and reports no problems with erection, ejaculation or pain with inter-
course. He has had no prior urogenital infections or exposure to sexually transmitted infections. He has had unprotected
sex prior to his current relationship, but has not knowingly conceived. He has no medical problems or past surgery. The
couple has vaginal intercourse 3-5 times per week when he is at home.
TH
APGO MEDICAL STUDENT EDUCATIONAL OBJECTIVES, 10 EDITION
TEACHING CASES

The female patient is 5’9” and weighs 130 pounds. Head and neck examination is unremarkable. Specifically there is no
evidence of thyromegaly. Breast exam reveals no tenderness or masses, but she has bilateral galactorrhea on compres-
sion of the areola. Pelvic exam reveals normal genitalia, well-estrogenized vaginal mucosa and cervical mucus consistent
with the proliferative phase. The uterus is anteflexed and normal in size without masses or tenderness. Several tests
were ordered.

COMPETENCY-BASED DISCUSSION & KEY TEACHING POINTS:

Competencies addressed:
• Patient Care
• Medical Knowledge
• Interpersonal and Communication Skills
• Professionalism

1. What is the definition of infertility?

• Inability to become pregnant despite 12 months of trying to conceive without using contraception
in women <35-years-old. Six months of unprotected intercourse defines infertility in women 35
years and older. About 15% of couples experience this problem.

2. What are the etiologies of infertility?

• Ovulatory dysfunction (20%) - anovulation


• Male factor (30%) – decreased sperm count, decreased motility or low normal forms (morphology)
• Tubal and pelvic (30%) – tubal damage due to pelvic infection, or pelvic factors such as endome-
triosis or pelvic adhesions
• Unexplained (15%)
• Unusual problems (5%)

Key Teaching Point: 1) Infertility has many etiologies. Often the etiology will involve issues with both part-
ners. 2) Addressing infertility often involves preconceptual testing and counselling for possible genetic disor-
ders.

3. What is the initial work-up for infertile couples and what tests would you add for this particular couple?

• Normal TSH
• Prolactin 60 ng/ml (normal range < 20 ng/ml)
• Evaluation for ovulation: Progesterone (day 21) was 1.2 ng/ml (≥ 3 ng/ml will indicate ovulation).
Could also review basal body temperature charting or have patient use ovulation predictor kits.
• Hysterosalpingogram demonstrated a normal uterine cavity with spill of radiopaque dye from
both fallopian tubes.
• Semen analysis with 2ml of semen (normal >1.5), 4 million sperm/ml (normal >15), 20% motil-
ity (normal >40%), 2% normal morphology (normal >4%).
• Discussion regarding frequency and timing of intercourse
• Ovarian reserve testing: Day 3 FSH 8.3 mIU/ml, estradiol <20 pg/ml, antimullerian hormone
(AMH) 1.1 ng/ml which are considered normal.

Copyright © 2014 by Association of Professors of Gynecology and Obstetrics (APGO).


For permissions: apgoadmin@apgo.org!
TH
APGO MEDICAL STUDENT EDUCATIONAL OBJECTIVES, 10 EDITION
TEACHING CASES

4. Offer genetic counseling and testing for cystic fibrosis mutations. Given the results of the tests, what is the differen-
tial diagnosis for the etiology(ies) of this couple’s infertility?

• Anovulation secondary to hyperprolactinemia from a potential prolactinoma


• Oligospermia – repeat semen analysis once and consider referral to a urologist.

5. What is the appropriate management for etiology of this couple’s infertility?

• For anovulation secondary to a possible prolactinoma, the patient should have a head MRI to
rule out a pituitary lesion. Treat with bromocriptine to lower prolactin levels, which will usually
result in regular ovulation.
• If she remains anovulatory after management of her prolactinoma with bromocriptine to nor-
malize her prolactin level, ovulation induction may be offered with clomiphene citrate.
• For oligospermia, refer to a urologist for evaluation for correctable causes. However, if oligo-
spermia remains after evaluation and treatment then options include in vitro fertilization with
intracytoplasmic sperm injection, intrauterine insemination with partner’s sperm, intrauterine
insemination with donor sperm, adoption.

6. The husband elects to undergo testing for common cystic fibrosis mutations and is determined to be a carrier.
What options are available to them to achieve a pregnancy that is less likely to lead to a child affected by cystic
fibrosis? Discuss the ethical issues associated with these choices.

• The couple could elect to use donor sperm or donor eggs. In both cases one of the parents
would not be genetic parents.
• The couple could elect to use IVF with preimplantation genetic diagnosis.
• The couple could elect to achieve a pregnancy with none of the above techniques and accept a
1:4 risk of having a child affected with CF. They could elect to undergo antepartum testing
(chorionic villi sampling, amniocentesis, etc.) to determine if the pregnancy is affected with CF.

REFERENCES

Beckman CRB, et al. Obstetrics and Gynecology. 7th ed. Philadelphia: Lippincott, Williams & Wilkins, 2013.

Hacker NF, Moore JG, et al. Essentials of Obstetrics and Gynecology. 5th ed. Philadelphia: Saunders, 2010.

Copyright © 2014 by Association of Professors of Gynecology and Obstetrics (APGO).


For permissions: apgoadmin@apgo.org!
TH
APGO MEDICAL STUDENT EDUCATIONAL OBJECTIVES, 10 EDITION
TEACHING CASES
!

U N I T 4: R E P R O D U C T I V E E N D O C R I N O L O G Y , I N F E R T I L I T Y &
RELATED TOPICS

Educational Topic 47: Menopause

Rationale: Women may spend much of their lives in the postmenopausal years. Physicians should understand the
physical and emotional changes caused by menopause.

Intended Learning Outcomes:!!

A student should be able to:

• Define menopause and describe changes in the hypothalamic-pituitary-ovarian axis associated with perimeno-
pause/menopause
• Describe symptoms and physical exam findings related to perimenopause/menopause
• Discuss management options for patients with perimenopause/menopausal symptoms
• Counsel patients regarding the menopausal transition
• Discuss long-term changes associated with menopause

TEACHING CASE

CASE: A 53-year-old, G3P3 woman, whose last menstrual period was 4 months ago presents to the office with hot
flushes, emotional lability, and insomnia. She experiences hot flushes 2-3 times per day and occasionally at night. She
has been having trouble sleeping and is extremely fatigued. Since age 14, her periods have been regular until 2 years
ago, when they began to space out to every 2-3 months. She is sexually active and recently has noted some dyspareunia.
The patient rarely exercises. She smokes 2 packs of cigarettes a day and drinks alcohol socially. She recently started
taking a soy supplement. She does not have any pertinent gynecological, medical or surgical history. Her family history
is significant for her mother sustaining a hip fracture at age 60 and a sister with breast cancer and high cholesterol. On
examination, she has normal vital signs. She is 5’4” tall and weighs 123 lbs. On pelvic examination, she has decreased
vaginal rugae and a pale, small cervix. No masses or tenderness are palpated on bimanual exam.

COMPETENCY-BASED DISCUSSION & KEY TEACHING POINTS:

Competencies addressed:
• Patient Care
• Medical Knowledge
• Interpersonal and Communication Skills
• Professionalism
• Systems-based Practice
TH
APGO MEDICAL STUDENT EDUCATIONAL OBJECTIVES, 10 EDITION
TEACHING CASES

1. What are the symptoms of perimenopause and menopause?

• Hypoestrogenism is the basis for the common changes of menopause.


• The common signs and symptoms of menopause include amenorrhea (of 12 months duration), hot flash-
es, memory changes, sleep difficulty, decreased libido, dyspareunia, urinary symptoms, breast changes.

2. How do you make the diagnosis of menopause?

• Menopause is the permanent cessation of menses and usually occurs between the ages of 50 and 55, with
an average of 50-52 years.
• The definition of menopause is the absence of menses for 12 consecutive months. It is, therefore, a retro-
spective diagnosis.
• Perimenopausal symptoms usually begin 3 to 5 years before amenorrhea or postmenopausal levels of
hormones.

3. What are the patient’s risk factors for osteoporosis?

• This patient’s risk factors include menopause, family history of osteoporosis, cigarette smoking, and sed-
entary lifestyle.
• Additional risk factors for discussion include age at menopause or oophorectomy, white or Asian origin,
small body frame or low BMI, high risk for osteoporosis related fracture per FRAX tool, vitamin D3 defi-
ciency, poor calcium intake, alcohol and caffeine intake, and corticosteroid use.

4. How do you diagnose and treat atrophic vaginitis?

• Patients commonly have vaginal dryness, vulvar irritation, pruritus, and dyspareunia. Associated urinary
symptoms may be present.
• Examination shows vulvar erythema. Excoriation may be present. Loss of vaginal rugae, a pale vaginal
mucosa, with patches of erythema and even superficial blood vessels are consistent with atrophy.
• The pale or yellow discharge has a pH of 5.5 or higher.
• Basal cells replace superficial vaginal epithelial cells and can be seen on a saline wet prep or Pap test.
• Treatment is topical estrogen (allow 4 to 6 weeks for symptomatic relief).

5. How do you counsel a patient regarding estrogen and alternative therapies?

• Risks and benefits of therapy should be reviewed (WHI and other studies).
• Contraindications should be discussed.
• Treatment options for menopausal symptoms and osteoporosis should be outlined.
• Bio-identical (compounded) hormones do not have an inherent advantage over standard therapies and
may vary in their potencies.
• Micronized progesterone and estradiol are bio-identical by definition.
• Any patient on systemic HT with an intact uterus needs a progestogen.
• Transdermal estrogen administration is preferable due to a beneficial effect on lipid balance and thrombo-
embolism risk.
• Lifestyle modifications including smoking cessation should be stressed.
• The importance of evaluating any postmenopausal bleeding should be discussed.
• Acknowledge frequent use of complementary and alternative treatments.
• SSRI antidepressants can be used as an alternative in women who are not candidates for HT.

Copyright © 2014 by Association of Professors of Gynecology and Obstetrics (APGO).


For permissions: apgoadmin@apgo.org!
TH
APGO MEDICAL STUDENT EDUCATIONAL OBJECTIVES, 10 EDITION
TEACHING CASES

• Various herbal supplements have been used for treatment of hot flashes, though they have not been
shown to be as effective as HT in placebo-controlled studies and concerns about safety have also been
raised.

6. What laboratory and diagnostic tests would you order for this patient?

• Laboratory and diagnostic tests should focus on the patient’s history and symptoms, as well as preventive
screening. For example, a TSH and lipid panel is appropriate given her fatigue and family history.
• General health maintenance/screening test guidelines (i.e. colonoscopy at age 50, bone density at age 65,
etc.) should be discussed. Tests include a mammogram, bone density (given patient’s smoking and family
history of fracture), colonoscopy.
• Discuss new cervical cytology screening recommendations.

REFERENCES

Beckman CRB, et al. Obstetrics and Gynecology. 7th ed. Philadelphia: Lippincott, Williams & Wilkins, 2013.

Hacker NF, Moore JG, et al. Essentials of Obstetrics and Gynecology. 5th ed. Philadelphia: Saunders, 2010.

Copyright © 2014 by Association of Professors of Gynecology and Obstetrics (APGO).


For permissions: apgoadmin@apgo.org!
TH
APGO MEDICAL STUDENT EDUCATIONAL OBJECTIVES, 10 EDITION
TEACHING CASES
!

U N I T 2: O B S T E T R I C S
SECTION B: ABNORMAL OBSTETRICS

Educational Topic 20:


Multifetal Gestation

Rationale: Multifetal gestation imparts additional risks and complications to the mother and fetus which requires
specialized care.

Intended Learning Outcomes:!!

A student should be able to:

• List the risk factors for multifetal gestation


• Describe embryology of multifetal gestation
• Describe the unique maternal and fetal physiologic changes associated with multifetal gestation
• Describe the diagnosis and management of multifetal gestation
• Describe the potential maternal and fetal complications associated with multifetal gestation

TEACHING CASE

CASE: You are seeing a 28 year-old G2P1 now at 12 weeks. Her first pregnancy was full term and uncomplicated. At
her first trimester screen she was noted to have a dichorionic diamniotic twin gestation with size equal to dates.

COMPETENCY-BASED DISCUSSION & KEY TEACHING POINTS:


Competencies addressed:
• Communication
• Patient Care
• Medical Knowledge
• Systems Based Practice

1. How is the diagnosis of chorionicity and zygosity made?


st
• 1 trimester ultrasound is the most accurate time to identify chorionicity.
• In addition to the identification of 2 placentas, membrane thickness and evaluation of the membrane in-
sertion site are also used to identify chorionicity
TH
APGO MEDICAL STUDENT EDUCATIONAL OBJECTIVES, 10 EDITION
TEACHING CASES

• Monozygous embryos dividing <72 hours after fertilization will be dichorionic (30% of monozygous
twins).
• Ultrasound diagnosis of dichorionic twins cannot determine zygosity.
• Monochorionic embryos dividing >72 hours after fertilization are always monozygous.

2. What nutritional deficiencies is she at higher risk for in a twin gestation? What recommendations will you make to
her because of them, including weight gain?

• The increased circulating blood volume of multiple gestations accentuates the dilutional anemia of preg-
nancy.
• Each fetus will extract Fe from maternal circulation further exacerbating the physiologic anemia.
• Calcium depletion is also exacerbated in multiple gestations.
• Normal weight woman are recommended to gain an additional 10-15 lbs (total 35-40).
• Calcium and iron supplementation should be recommended even prior to anemia.

3. You are counseling her about the increased maternal and fetal risks during the pregnancy, what specifically are you
concerned about?

• Maternal risks include increased incidence of gestational diabetes, hypertension, anemia as well as ante
and postpartum hemorrhage.
• There is an increased incidence of thrombosis, compounded by the increased risks of obesity, maternal
age, bed rest and Cesarean deliveries in multiple gestations.
• Fetal risks include an increased chance of miscarriage, fetal growth restriction, preterm delivery, perinatal
asphyxia and stillbirth (of one or both). All are more common in monochorionic gestations.
• The risk of fetal anomalies is more common in all multiple gestations, but each of a dichorionic twin set
has the same risk of structural anomalies as a singleton. The risk to a fetus of a monochorionic gestation is
double a singletons baseline risk.

4. What additional management strategies are recommended in twin pregnancy?

• More frequent prenatal visits to screen for maternal hypertension.


• Periodic ultrasound surveillance to screen for fetal growth.
• Serial cervical ultrasound has been shown to be able to predict preterm delivery in twins to allow time for
betamethasone use.
• Antenatal fetal testing is generally recommended in later pregnancy to evaluate increased fetal risk of con-
tinuing pregnancy.

5. Your patient is now at 29 weeks without any complications. You are going to counsel her about delivery planning.
What factors will determine the safest timing of delivery in a multiple gestation?

• 38 weeks has been shown to have the lowest risk of perinatal mortality in uncomplicated twin gestations.
• Maternal or fetal complications of pregnancy may warrant safest delivery at an earlier gestational age.

6. What are the risks of delivery in a multiple gestation and what are considerations for mode of delivery?

• Increased fetal risks include perinatal asphyxia, birth trauma; both primarily to the second twin.
• Discussion of mode of delivery needs to include fetal presentation, fetal and maternal status and time of
delivery and ability to monitor both fetuses reliably.

Copyright © 2014 by Association of Professors of Gynecology and Obstetrics (APGO).


For permissions: apgoadmin@apgo.org!
TH
APGO MEDICAL STUDENT EDUCATIONAL OBJECTIVES, 10 EDITION
TEACHING CASES

• Maternal risks include increased risk of Cesarean delivery, postpartum hemorrhage, and anesthesia com-
plications.

REFERENCES

Beckman CRB, et al. Obstetrics and Gynecology. 7th ed. Philadelphia: Lippincott, Williams & Wilkins, 2013.

th
Hacker NF, Moore JG, et al. Essentials of Obstetrics and Gynecology. 5 ed. Philadelphia: Saunders, 2010.

Bush MC, Pernoll ML. Multiple Pregnancy: in Current Diagnosis & Treatment: Obstetrics and Gynecology, 2007.

ACOG Practice Bulletin 56, Multiple Gestation: Complicated Twin, Triplet and Higher Order Multifetal Gestation,
2004.

Chauhan SP, Cardo JA, Hayes E. Twins: prevalence, problems, and preterm births. Am J Obstet Gynecol: 203(4): 305-
315. 2010.

Copyright © 2014 by Association of Professors of Gynecology and Obstetrics (APGO).


For permissions: apgoadmin@apgo.org!
TH
APGO MEDICAL STUDENT EDUCATIONAL OBJECTIVES, 10 EDITION
TEACHING CASES
!

U N I T 1: A P P R O A C H T O T H E P A T I E N T

Educational Topic 3: Pap Test and DNA


Probes/Cultures

Rationale: The Pap smear is one of the most effective screening tests used in medicine today. Proper technique in
performing the Pap smear and obtaining specimens for DNA probes and/or microbiologic culture will improve
accuracy.

Intended Learning Outcomes:!!

A student should be able to:

• Perform a Pap smear


• Obtain specimens to detect sexually transmitted infections
• Explain the purpose of these tests to the patient

TEACHING CASE

CASE: JA is a 24-year-old G0 female who presents for an annual exam. She had a normal Pap smear reading with no
intraepithelial lesion three years ago. She is currently sexually active with a new partner for the last 2 months, and has
had four lifetime partners. She does not report any history of sexually transmitted diseases. She is currently using birth
control pills for contraception. She is just finishing her menstrual cycle and is spotting; her periods are regular without
problems. Her review of systems is negative. She smokes one pack of cigarettes per week; she does not drink and states
that she has occasionally used marijuana. Her vitals and physical exam are normal.

COMPETENCY-BASED DISCUSSION & KEY TEACHING POINTS:


Competencies addressed:
• Patient Care
• Medical Knowledge
• Interpersonal and Communication Skills
• Professionalism

1. How are cervical samples for pap smears collected?

• Select contoured end of plastic spatula and rotate it 360 degrees around the entire exocervix while main-
taining tight contact with exocervical surface. Then swirl the spatula vigorously in the vial 10 times
TH
APGO MEDICAL STUDENT EDUCATIONAL OBJECTIVES, 10 EDITION
TEACHING CASES

• Insert the endo-brush into the cervix until only the bottommost fibers are exposed. Slowly rotate 1/4 or
1/2 turn in one direction. DO NOT OVER-ROTATE. Then swirl the spatula vigorously in the vial 10
times.

2. What are the different laboratory techniques available to perform a pap smear?

• Discuss conventional Pap smear and Thin Prep: Both are collected from the endocervical canal and trans-
formation zone with spatula and cytobrush or cytobroom. Thin prep is the most widely used technique.
• Discuss the importance of properly labeling specimens, including patient’s LMP and sending it off to the
lab promptly.

3. In addition to performing a Pap smear, what other tests might you recommend for this patient, and how do you
proceed to obtain them?

• Since the patient is sexually active with a new partner, you should offer her screening for sexually trans-
mitted diseases including gonorrhea and chlamydia. All sexually active patients under the age of 25 should
be screened annually for chlamydia and gonorrhea. Screening for gonorrhea and chlamydia does not re-
quire a speculum exam as they can be tested from a urine sample.
• To collect an endocervical sample, wipe any excess cervical mucous and insert the DNA probe in the en-
docervical canal.

4. How often does this patient need to undergo a Pap smear?

• A patient who is between the ages of 21-30 only needs a pap test every three years. Patients who have had
an abnormal Pap smear previously will need more frequent Pap smears based on the actual abnormality.,
This patient needs to continue to have Pap smears every 3 years, assuming this one is normal.

5. What risk factors does this patient have for cervical dysplasia and cancer?

• Multiple partners, sexual activity at an early age, cigarette smoking.

6. If while performing the speculum exam, you notice a gross lesion on the cervix, what would your next step in man-
agement be?

• Any gross lesion on the cervix will need to be directly biopsied as Pap smears have a false negative rate
and cervical cancer might be missed.

7. What other counseling or advice do you need to discuss with this patient?

• Discuss the importance of cervical cancer screening guidelines. Offer the patient blood testing for other
sexually transmitted infections such as syphilis, HIV, and hepatitis B and C. Discuss the importance of safe
sex practices including condom use.
• Discuss the indications and risks/benefits of an HPV vaccine such as Gardasil. HPV vaccination is rec-
ommended for all females between 9 and 26 to reduce the incidence of cervical cancer.

Copyright © 2014 by Association of Professors of Gynecology and Obstetrics (APGO).


For permissions: apgoadmin@apgo.org!
TH
APGO MEDICAL STUDENT EDUCATIONAL OBJECTIVES, 10 EDITION
TEACHING CASES

REFERENCES

Beckman CRB, et al. Obstetrics and Gynecology. 7th ed. Philadelphia: Lippincott, Williams & Wilkins, 2013.

Hacker NF, Moore JG, et al. Essentials of Obstetrics and Gynecology. 5th ed. Philadelphia: Saunders, 2010.

ACOG Practice Bulletin 131, Screening for Cervical Cancer, November 2012.

Copyright © 2014 by Association of Professors of Gynecology and Obstetrics (APGO).


For permissions: apgoadmin@apgo.org!
TH
APGO MEDICAL STUDENT EDUCATIONAL OBJECTIVES, 10 EDITION
TEACHING CASES
!

U N I T 2: O B S T E T R I C S
SECTION B: ABNORMAL OBSTETRICS

Educational Topic 27:


Postpartum Hemorrhage

Rationale: Postpartum hemorrhage is a major, often preventable, cause of maternal morbidity and mortality.

Intended Learning Outcomes:!!

A student should be able to:

• List the risk factors for postpartum hemorrhage


• Construct a differential diagnosis for immediate and delayed postpartum hemorrhage
• Develop an evaluation and management plan for the patient with postpartum hemorrhage including consider-
ation of various resource settings

TEACHING CASE

CASE: Tracy is a 33 year-old G1 woman who underwent induction of labor for a post-dates pregnancy at 41 weeks and
3 days gestation. Prostaglandins were used to accomplish cervical ripening and an oxytocin infusion was used to induce
labor. The patient had a lengthy first and second stage. Ultimately, the fetus was delivered with vacuum assistance. The
baby weighed 9 pounds 3 oz at birth. The third stage of labor was uncomplicated. Thirty minutes later you are called to
the recovery room because Tracy has experienced brisk vaginal bleeding that did not respond to uterine massage by her
nurse.

COMPETENCY-BASED DISCUSSION & KEY TEACHING POINTS:


Competencies addressed:
• Patient Care
• Medical Knowledge
• Systems-Based Practice

1. What is the definition of postpartum hemorrhage?

• Varying definitions among sources.


• Blood loss greater than or equal to 500 ml within 24 hours after vaginal delivery or 1000 mL after Cesare-
an delivery.
• Decline in hematocrit of more than 10% may be used.
TH
APGO MEDICAL STUDENT EDUCATIONAL OBJECTIVES, 10 EDITION
TEACHING CASES

• Severe PPH is greater than or equal to 1000 ml within 24 hours after delivery.
• Primary postpartum hemorrhage occurs in first 24 hours, while secondary hemorrhage occurs after 24
hours and before 6-12 weeks following delivery.

Key Teaching Point: Postpartum hemorrhage is the leading cause of maternal death worldwide. In most cases
it is preventable. At present, there is no good way to estimate blood loss quantitatively over clinical estimation.

2. What elements of this case present risk factors for a postpartum hemorrhage?

• Induced labor
• Prolonged labor
• Operative vaginal delivery
• Fetal macrosomia

3. What are other risk factors for postpartum hemorrhage?

• Multiparity
• Over-distended uterus (multiples, hydramnios, fetal macrosomia)
• Augmented labor
• Prolonged labor
• Operative delivery
• Previous history of postpartum hemorrhage
• Chorioamnionitis

Key Teaching Point: There are many risk factors for postpartum hemorrhage; however, there are times when
no risk factor for PPH can be identified.

4. What are the causes of postpartum hemorrhage?

• Uterine atony
• Retained placental tissue
• Maternal trauma/obstetric lacerations
• Uterine inversions
• Maternal coagulopathy (pre-existing or acquired)

Key Teaching Point: The causes of postpartum hemorrhage can be remembered by the 4 T’s (Tone, Tissue,
Trauma and Thrombin). Learners should be aware that uterine atony is by far the most common cause of
postpartum hemorrhage.

5. What is the management for postpartum hemorrhage?

• Prevention:
• Active Management of the third stage of labor
• Oxytocin (IV or IM) with delivery of anterior shoulder or delivery of the fetus
• Gentle cord traction following delivery of fetus
• Suprapubic support of the uterus to prevent inversion while providing cord traction

Copyright © 2014 by Association of Professors of Gynecology and Obstetrics (APGO).


For permissions: apgoadmin@apgo.org!
TH
APGO MEDICAL STUDENT EDUCATIONAL OBJECTIVES, 10 EDITION
TEACHING CASES

• Diagnosis of PPH and Management


• ABC
• Assess tone of uterus
• Bimanual massage
• Employ uterotonics (oxytocin, ergonovine/methyl-ergonovine, 15-methyl prostaglandin F2α,
misoprostol)
• Empty bladder, insert foley catheter for fluid monitoring
• If uterus does not respond to these methods consider alternatives measures (intrauterine com-
pression, surgery with compression sutures, arterial ligation, hysterectomy)
• If uterine tone is adequate consider other etiologies
• Retained placental fragments
• Consider curettage
• Maternal laceration of lower genital tract
• Consider examination under anesthesia and repair
• Maternal Coagulopathy
• Consider involvement of other specialties such as hematology
• Give appropriate factor replacement
• Identify underlying cause
• Infection, hemorrhage, amniotic fluid embolism

Key Teaching Point: Postpartum hemorrhage is a sign, not a diagnosis. It is critical to identify the underlying
cause rapidly so that it can be remedied. Management of a severe postpartum hemorrhage should include
members of the health care team including obstetrics, nursing, anesthesia, hematology, internal medicine,
transfusion medicine and intensive care specialists. Protocols should be in place to deal with postpartum hem-
orrhage and simulations should be undertaken to ensure all members of the team are well versed in the man-
agement of this potentially fatal obstetrical complication.

REFERENCES

Beckman CRB, et al. Obstetrics and Gynecology. 7th ed. Philadelphia: Lippincott, Williams & Wilkins, 2013.

Hacker NF, Moore JG, et al. Essentials of Obstetrics and Gynecology. 5th ed. Philadelphia: Saunders, 2010.

ACOG Practice Bulletin Number 76, Postpartum Hemorrhage, 2006; Reaffirmed 2013.

SOGC Practice Guideline 235, Active Management of the Third Stage of Labour: Prevention and Treatment of Postpar-
tum Hemorrhage, October 2009.

World Health Organization Guidelines for the management of postpartum hemorrhage and retained placenta, 2009.

Copyright © 2014 by Association of Professors of Gynecology and Obstetrics (APGO).


For permissions: apgoadmin@apgo.org!
TH
APGO MEDICAL STUDENT EDUCATIONAL OBJECTIVES, 10 EDITION
TEACHING CASES
!

U N I T 2: O B S T E T R I C S
SECTION B: ABNORMAL OBSTETRICS

Educational Topic 18:


Preeclampsia-Eclampsia

Rationale: Preeclampsia-eclampsia accounts for significant morbidity and mortality in both the mother and newborn.

Intended Learning Outcomes:!!

A student should be able to:

• Define the types of hypertension in pregnancy


• Describe the pathophysiology of preeclampsia-eclampsia
• List risk factors for preeclampsia
• Recognize the signs and symptoms to diagnose preeclampsia-eclampsia
• Explain the management of a patient with preeclampsia-eclampsia
• List the maternal and fetal complications associated with preeclampsia-eclampsia

TEACHING CASE

CASE: An 18 year old G1P0 currently at 38 0/7 weeks presents for her routine prenatal visit. She has had an uncompli-
2
cated pregnancy up to this point, with the exception of a late onset of prenatal care and obesity (BMI of 35 kg/m ). She
reports that during the past week, she has noted some swelling of her hands and feet. She also has been feeling a bit
more fatigued and has had a headache on and off. She reports good fetal movement. She has had some contractions on
and off, but nothing persistent. Her blood pressure is 147/92 and her urine dip has 1+ protein/no ketones/no glucose.
The fundal height measures 36 cm, the fetus is cephalic with a heart rate of 144 bpm. On physical exam you note that
the patient has 3+ pre-tibial edema, and trace edema of her hands and face. She has 2+ deep tendon reflexes and 2 beats
of clonus. You review her blood pressures up to this point and note that at the time of her first prenatal visit at 18
weeks, her blood pressure was 130/76 and she had no protein in her urine. However, since that visit, her blood pres-
sures seem to have been climbing higher with each visit. Her last visit was one week ago, and she had a blood pressure
of 138/88 with trace protein in the urine and she has gained 5 pounds.
TH
APGO MEDICAL STUDENT EDUCATIONAL OBJECTIVES, 10 EDITION
TEACHING CASES

COMPETENCY-BASED DISCUSSION & KEY TEACHING POINTS:


Competencies addressed:
• Patient care
• Medical Knowledge
• System-Based Practice

1. What is considered a hypertensive blood pressure during pregnancy?

• In pregnancy, hypertension is defined as either a systolic blood pressure ≥ 140 or diastolic blood pressure ≥
90 or both.

2. What types of hypertensive syndromes can occur during pregnancy?

• Chronic hypertension: Requires that the patient have documented hypertension preceding 20 weeks
gestation, or where hypertension is first noted during pregnancy and persists for longer than 12 weeks
postpartum
• Preeclampsia-eclampsia: Development of new onset hypertension and proteinuria after 20 weeks of
pregnancy. Is stratified into mild and severe forms. There are atypical forms of preeclampsia as well.
• Preeclampsia superimposed on chronic hypertension: Superimposed preeclampsia should be reserved for those
women with chronic hypertension who develop new-onset proteinuria (≥ 300 mg in a 24-hour collection)
th
after the 20 week of pregnancy. In pregnant women with preexisting hypertension and proteinuria, the
diagnosis of superimposed preeclampsia should be considered if the patient experiences sudden significant
increases in blood pressure or proteinuria or any of the other signs and symptoms consistent with severe
preeclampsia.
• Gestational Hypertension: Hypertension without proteinuria which first appears after 20 weeks gestation or
within 48 to 72 hours after delivery and resolves by 12 weeks postpartum.

3. How does the physiology of preeclampsia lead to the clinical symptoms and findings?

• Hypoxia, hypoperfusion and ischemia lead to the clinical placental pathophysiology (with fetal
compromise: IUGR, oligohydramios, placental abruption)
• Systemic endothelial dysfunction leads to central & peripheral edema, proteinuria, and hypertension
(from disruption of vascular regulation). Endothelial dysfunction in target organs leads to headache,
epigastric pain, and renal dysfunction. Microvascular endothelial destruction leads to release of
procoagulants and DIC.

4. What are the laboratory findings that support a diagnosis of preeclampsia-eclampsia syndrome?

• Proteinuria (> 300 mg on a 24 hour urine collection)


• Elevated hematocrit
• Hemolysis
3
• Thrombocytopenia (< 100,000 cells/mm )
• Elevated liver enzymes (ALT/AST twice normal)
• Elevated serum uric acid concentration

Copyright © 2014 by Association of Professors of Gynecology and Obstetrics (APGO).


For permissions: apgoadmin@apgo.org!
TH
APGO MEDICAL STUDENT EDUCATIONAL OBJECTIVES, 10 EDITION
TEACHING CASES

5. What types of maternal and fetal complications are associated with preclampsia-eclampsia syndrome?

• Maternal:
! CNS: eclamptic seizure, stroke
! Cardiopulmonary: pulmonary edema
! Hepatic: Subcapsular hematoma or hepatic rupture
! Renal: renal failure or acute tubular necrosis
! Hematologic: hemorrhage, DIC
• Fetal:
! Preterm delivery
! Abruptio Placenta
! Fetal growth restriction
! Hypoxic ischemic encephalopathy
! Fetal death

REFERENCES

Beckman CRB, et al. Obstetrics and Gynecology. 7th ed. Philadelphia: Lippincott, Williams & Wilkins, 2013.

Hacker NF, Moore JG, et al. Essentials of Obstetrics and Gynecology. 5th ed. Philadelphia: Saunders, 2010.

Sibai B. Diagnosis and Management of Gestational Hypertension and Preeclampsia. ObstetGynecol. High-Risk Preg-
nancy Series: An Expert's View. Jul 2003; (102) 1 - p 181-192.

Lain KY, Roberts JM. Contemporary concepts of the pathogenesis and management of preeclampsia. JAMA. Jun
26 2002; 287(24):3183-6.

Copyright © 2014 by Association of Professors of Gynecology and Obstetrics (APGO).


For permissions: apgoadmin@apgo.org!
TH
APGO MEDICAL STUDENT EDUCATIONAL OBJECTIVES, 10 EDITION
TEACHING CASES
!

U N I T 2: O B S T E T R I C S
SECTION B: ABNORMAL OBSTETRICS

Educational Topic 24:


Preterm Labor

Rationale: Prematurity is one of the most common cause of neonatal morbidity and mortality. The reduction of
preterm births remains an important goal in obstetric care.

Intended Learning Outcomes:!!

A student should be able to:

• Identify the modifiable and non-modifiable risk factors and causes for preterm labor
• Describe the signs and symptoms of preterm labor
• Describe the initial management of preterm labor
• List indications and contraindications of medications used in preterm labor
• List the adverse outcomes associated with preterm birth
• Describe the counseling for reducing preterm birth risk

TEACHING CASE

CASE: An 18-year-old African-American, G2P0101 woman who is 12 weeks pregnant, presents to your prenatal clinic
for a new patient visit. Before you walk into the room to see the patient, you look through her records and note that she
delivered her last pregnancy just 12 months ago. Beginning at 24 weeks in her previous pregnancy, the patient present-
ed numerous times to Labor and Delivery reporting contractions, and was sent home each time with a diagnosis of
“Braxton-Hicks contractions.” She eventually presented at 28 weeks gestation and was diagnosed with preterm labor.
She delivered at 29 weeks. The neonate’s course was complicated by intra-ventricular hemorrhage and respiratory dis-
tress syndrome. The child now appears to have cerebral palsy and chronic lung disease due to bronchopulmonary dys-
plasia.

COMPETENCY-BASED DISCUSSION & KEY TEACHING POINTS:


Competencies addressed:
• Patient care
• Medical knowledge
• Interpersonal and communication skills
• Professionalism
• Systems-based practice and management
TH
APGO MEDICAL STUDENT EDUCATIONAL OBJECTIVES, 10 EDITION
TEACHING CASES

1. What are the risk factors for preterm labor, and which ones does this patient have?

• The diagnosis of preterm labor generally is based on clinical criteria of regular uterine contractions ac-
companied by a change in cervical dilation, effacement, or both or initial presentation with regular con-
tractions and cervical dilation of at least 2 cm between 20 0/7 - 36 6/7 weeks of gestation. Preterm birth
related to spontaneous preterm labor should be distinguished from preterm birth that is iatrogenic due to
maternal or fetal complications. Risk factors for spontaneous preterm labor include:
• Prior history of preterm birth (highest risk)
• African-American race
• Low pre-pregnancy BMI
• Preterm uterine contractions
• Premature rupture of membranes (PROM)
• Incompetent cervix
• Shortened cervix on transvaginal ultrasound
• Infections
! Urinary
! Vaginal (BV)
! Intra-amniotic
• Excessive uterine enlargement
! Polyhydramnios
! Multiple gestation
• Uterine distortion
! Leiomyomas
! Septate uterus, uterine didelphis, and other anomalies
• Placental abnormalities
! Abruption placentae
! Placenta previa
• Maternal smoking (associated with PROM)
• Substance abuse
• Inflammation
• Decidual hemorrhage
• Pathologic uterine distension
• This patient history is notable for a number of risk factors for preterm labor, namely:
• Prior history of preterm birth
• African-American race
• Low maternal weight
• Short interpregnancy interval
• The physiology of labor initiation is not completely understood. Once fetal maturity is reached, there
is activation of the maternal and fetal hypothalamus/pituitary/ adrenal axis that leads to the initiation
of parturition. It has been proposed that fetal inflammatory response can contribute to the onset of
preterm labor.

2. What characteristics distinguish Braxton-Hicks contractions from true labor contractions?

• Braxton-Hicks contractions: The uterus undergoes irregular and sporadic contractions that are usually pain-
less or of mild intensity. The frequency of these contractions increases in the last 4 to 8 weeks of pregnan-

Copyright © 2014 by Association of Professors of Gynecology and Obstetrics (APGO).


For permissions: apgoadmin@apgo.org!
TH
APGO MEDICAL STUDENT EDUCATIONAL OBJECTIVES, 10 EDITION
TEACHING CASES

cy. These contractions are not associated with progressive cervical dilation and effacement, and often is
referred to as false labor. These contractions often resolve with rest, hydration, and/or sedation.
• True Labor Contractions: As compared to Braxton-Hicks contractions, true labor contractions occur at
regular intervals, progressively increase in frequency and intensity, and are associated with cervical dila-
tion. True labor contractions will not resolve with sedation.

3. What should you counsel the patient regarding the signs and symptoms of preterm labor?

• Distinguishing true preterm labor from false labor or Braxton-Hicks contractions is challenging and de-
pends primarily on prompt patient evaluation. Efforts to develop models or tests to predict preterm labor
have thus far been unsuccessful; therefore, when in doubt it is advisable to have patient come in for evalu-
ation. The primary method for identifying preterm labor is by screening for maternal signs and symptoms
as summarized below:
• Menstrual-like cramps
• Low, dull backache
• Abdominal pressure
• Pelvic pressure
• Abdominal cramping (with or without diarrhea)
• Increase or change in vaginal discharge (mucous, watery, light bloody discharge)
• Uterine contractions, often painless

4. What recommendations, if any, would you discuss with this patient regarding prevention strategies to reduce the
risk of preterm delivery in this pregnancy? To reduce the risk of neurodevelopmental disorders and other morbidi-
ty associated with preterm labor in this fetus should she experience preterm labor?

• Intramuscular or vaginal progesterone, begun in the second trimester, has been shown to decrease the
rate of preterm birth in women who previously experienced a preterm birth. Evidence does not currently
exist to recommend one route of progesterone delivery over the other in preventing preterm delivery.
• Magnesium sulfate administered to the mother, prior to an anticipated preterm birth, reduces the risk of
cerebral palsy in surviving infants; thus if delivery before 34 weeks gestation is anticipated, consideration
should be given to administration of magnesium sulfate.
• Antibiotics should be administered during preterm labor as prophylaxis against Group B strep sepsis in
the neonate.
• A course of antenatal steroids (betamethasone or dexamethasone) should be administered to the mother
diagnosed with preterm labor in an effort to enhance fetal lung maturity and decrease the risk of necrotiz-
ing fasciitis and intracranial bleeds in the neonate.

5. If the patient does experience PTL in this pregnancy, what recommendations would you make regarding treatment
and management?

• Fetal fibronectin testing (negative) and cervical length (greater than 2.5 cm) have good negative predictive
value in deciding which patients do not require treatment for preterm labor.
• There is no clear first line tocolytic medication; however, current medications in use include magnesium
sulfate, nifedipine, indomethacin, and beta-mimetics. The primary benefit of tocolytics appears to be de-
laying delivery in order to complete a course of antenatal steroids and transport the mother to a tertiary
care facility for adequate care of the premature newborn. The risks of tocolytics include:
• Magnesium (maternal flushing, decreased reflexes, muscle weakness, pulmonary edema and fetal
lethargy, hypotonia, respiratory distress and bone abnormalities if used > 7 days)
Copyright © 2014 by Association of Professors of Gynecology and Obstetrics (APGO).
For permissions: apgoadmin@apgo.org!
TH
APGO MEDICAL STUDENT EDUCATIONAL OBJECTIVES, 10 EDITION
TEACHING CASES

• Nifedipine (maternal hypotension)


• Indomethacin (maternal nausea, esophageal reflux, gastritis, emesis and possible platelet dysfunction
and fetal in utero closure of ductus arteriosus with > 48 hour use and possibly patent ductus arterio-
sus for neonate)
• Beta-mimetics (maternal tachycardia, hypotension, tremor, shortness of breath, chest discomfort,
pulmonary edema, hypokalemia and hyperglycemia and fetal tachycardia)

6. What are the potential adverse outcomes of preterm birth for the fetus?

• Respiratory distress syndrome


• Intraventricular hemorrhage
• Necrotizing enterocolitis
• Sepsis
• Neurologic impairment
• Seizures
• Long term-broncholpulmonary dysplasia
• Developmental abnormalities including cerebral palsy

REFERENCES

Beckman CRB, et al. Obstetrics and Gynecology. 7th ed. Philadelphia: Lippincott, Williams & Wilkins, 2013.

th
Hacker NF, Moore JG, et al. Essentials of Obstetrics and Gynecology. 5 ed. Philadelphia: Saunders, 2010.
th
Gabbe S, et al. Obstetrics: Normal and Problem Pregnancies. 6 ed. Philadelphia: Saunders, 2012.

ACOG Practice Bulletin 127, June 2012.

ACOG Practice Bulletin 130, October 2012.

ACOG Committee Opinion 455, reaffirmed 2013.

ACOG Committee Opinion 120, June 2011.

Copyright © 2014 by Association of Professors of Gynecology and Obstetrics (APGO).


For permissions: apgoadmin@apgo.org!
TH
APGO MEDICAL STUDENT EDUCATIONAL OBJECTIVES, 10 EDITION
TEACHING CASES
!

U N I T 2: O B S T E T R I C S
SECTION B: ABNORMAL OBSTETRICS

Educational Topic 25:


Premature Rupture of Membranes

Rationale: Rupture of the membranes prior to labor is a problem for both term and preterm pregnancies. Careful
evaluation and management of this condition may improve fetal and maternal outcome.

Intended Learning Outcomes:!!

A student should be able to:

• List the history, physical findings and diagnostic methods to confirm rupture of the membranes
• Identify risk factors for premature rupture of the membranes
• Describe the risks and benefits of expectant management versus immediate delivery based on gestational age
• Describe the methods to monitor maternal and fetal status during expectant management

TEACHING CASE

CASE: A 26-year-old G2P0100 woman, who is 31 weeks gestation, presents to the labor unit complaining of leakage of
fluid and she thinks that her “bag of water broke.” She has had increased vaginal discharge and intermittent lower back
pain for the last two days. She reports a gush of fluid about 2 hours ago. The fluid ran down her leg and appeared clear
with no noticeable odor. Her prior pregnancy was complicated by preterm labor and premature rupture of the
membranes at 26 weeks gestation. The neonate’s course was complicated by necrotizing enterocolitis, respiratory
distress, and death at 28 days of life.

COMPETENCY-BASED DISCUSSION & KEY TEACHING POINTS:


Competencies addressed:
• Patient care
• Medical knowledge

1. What risk factors are associated with premature rupture of membranes (PROM)?

• The definition of PROM is rupture of membranes before the onset of labor. Membrane rupture before la-
bor and before 37 weeks of gestation is referred to as preterm PROM.
• Risk factors for PROM include similar risks for preterm labor:
! Vaginal, cervical and intraamniotic infections
TH
APGO MEDICAL STUDENT EDUCATIONAL OBJECTIVES, 10 EDITION
TEACHING CASES

! Prior PROM
! Prior preterm delivery
! Low socioeconomic status
! Second and third-trimester bleeding
! Low body mass index
! Cervical insufficiency
! Cervical conization/LEEP
! Connective tissue disorders (Ehlers-Danlos syndrome)
! Nutritional deficiencies of copper and ascorbic acid
! Maternal cigarette smoking
! Illicit drug use
! Pulmonary disease in pregnancy
! Uterine overdistension
! Amniocentesis

2. What should be the next step in this patient’s diagnosis?

• Confirmation of the diagnosis of PROM.


! Sterile speculum examination to confirm the diagnosis
! Pooling of fluid per cervical os
! Fern – cervical mucus broad fern vs. amniotic fluid narrow fern
! pH (Nitrazine) – turns blue as the pH of amniotic fluid is usually 7.1-7.3.
! False positive Nitrazine may occur due to
! Alkaline urine
! Semen
! Blood
! Cervical mucus
! Antiseptic solutions
! Bacterial vaginosis
! Ultrasound evaluation AFI in equivocal cases – not diagnostic
! Test kits for amniotic proteins - considered ancillary to standard methods of diagnosis

3. What should be the next step in management once PROM has been confirmed?

• Assess fetal status: continuous fetal monitoring, ultrasound to assess the estimated fetal weight (EFW),
amniotic fluid volume and fetal presentation
• Rule out labor (uterine activity monitoring)
• Rule out intraamniotic infection: This diagnosis may be made clinically. In some cases amniocentesis may
prove helpful to rule out an intraamniotic infection. Amniotic fluid may be sent for gram stain, aerobic
and anaerobic cultures, glucose and cell count.
• Obtain swabs to rule out Chlamydia trachomatis, Neisseria gonorrhea and group B streptococcal infection
• Digital cervical examinations should be avoided unless the patient appears to be in active labor or immi-
nent delivery is planned. Digital exams are associated with an increased risk of infection and add little in-
formation to that available with speculum examination. Sterile speculum examination provides an
opportunity to confirm the diagnosis of PROM, inspect for cervicitis and umbilical cord or fetal prolapse,
assess cervical dilatation and effacement, and obtain cultures as appropriate.
• Once labor and intraamniotic infection have been ruled out, if patient is preterm (< 34 weeks) consider:

Copyright © 2014 by Association of Professors of Gynecology and Obstetrics (APGO).


For permissions: apgoadmin@apgo.org!
TH
APGO MEDICAL STUDENT EDUCATIONAL OBJECTIVES, 10 EDITION
TEACHING CASES

! Antibiotics: Ampicillin and erythromycin to prolong the latency period


! Steroids to enhance fetal lung maturation and decrease risk of RDS
! Patients with preterm PROM at a viable gestational age should be observed closely in the hospital on
modified bedrest. They should be assessed periodically for evidence of infection, placental abruption,
umbilical cord compression, fetal well-being, and labor. There is no consensus on the optimal
frequency and type of assessment that is optimal. An acceptable strategy would include periodic
ultrasound monitoring of amniotic fluid volume and daily or twice-daily fetal heart rate monitoring.
• The decision to deliver the fetus is based on gestational age and fetal status.
! If there is evidence of intraamniotic infection or evidence of fetal compromise at any gestational age,
the fetus should be delivered.
! The timing of delivery may vary among institutions:
! The patient who experiences PROM between 24 weeks and 31 completed weeks of gestation
should be cared for expectantly if no maternal or fetal contraindications exist until approximately
34 weeks of gestation.
! At 32–33 completed weeks of gestation, the risk of severe complications of prematurity is low if
fetal pulmonary maturity is confirmed by amniotic fluid samples collected vaginally or by
amniocentesis. Therefore, labor induction may be considered if pulmonary maturity has been
documented. If pulmonary maturity cannot be established, expectant management may be
beneficial.

4. What are the risks associated with preterm PROM?

• Maternal risks:
! Chorioamnionitis
! Cesarean delivery for malpresentation and failed induction
! Abruption
• Fetal risks:
! Cord prolapse
! Respiratory Distress Syndrome
! Necrotizing Enterocolitis (NEC)
! Infection (sepsis)
! Intraventricular hemorrhage- The risk for this varies with gestational age.
! Pulmonary hypoplasia especially if < 19 weeks when PROM occurs (rare after 26 weeks gestation)
! Skeletal deformities

5. What treatment can this patient be offered in a future pregnancy to decrease her recurrence risk for preterm
PROM and preterm delivery?

• Recent studies have suggested progesterone therapy to reduce the risk of recurrent spontaneous preterm
birth resulting from preterm labor or PROM.

Copyright © 2014 by Association of Professors of Gynecology and Obstetrics (APGO).


For permissions: apgoadmin@apgo.org!
TH
APGO MEDICAL STUDENT EDUCATIONAL OBJECTIVES, 10 EDITION
TEACHING CASES

REFERENCES

Beckman CRB, et al. Obstetrics and Gynecology. 7th ed. Philadelphia: Lippincott, Williams & Wilkins, 2013.

Hacker NF, Moore JG, et al. Essentials of Obstetrics and Gynecology. 5th ed. Philadelphia: Saunders, 2010.

ACOG Technical Bulletin 139, Premature Rupture of Membranes, October 2013.

Copyright © 2014 by Association of Professors of Gynecology and Obstetrics (APGO).


For permissions: apgoadmin@apgo.org!
TH
APGO MEDICAL STUDENT EDUCATIONAL OBJECTIVES, 10 EDITION
TEACHING CASES
!

U N I T 3: G Y N E C O L O G Y
SECTION A: GENERAL GYNECOLOGY

Educational Topic 36: Sexually


Transmitted Infections (STI) and Urinary
Tract Infections (UTI)

Rationale: Early recognition and treatment of urinary and pelvic infections may help prevent short and long-term
morbidity. Prevention of sexually transmitted infections is a major public health goal.

Intended Learning Outcomes:!!

A student should be able to:

• Describe the guidelines for STI screening and partner notification/treatment


• Describe STI prevention strategies, including immunization
• Describe the symptoms and physical exam findings associated with common STIs
• Discuss the steps in the evaluation and management of common STIs including appropriate referral
• Describe the pathophysiology of salpingitis and pelvic inflammatory disease
• Describe the evaluation, diagnostic criteria and initial management of salpingitis/pelvic inflammatory disease
• Identify possible long-term sequelae of salpingitis/pelvic inflammatory disease
• Describe the diagnosis and management of UTIs

TEACHING CASE

CASE: A 16-year-old G1P1 female, LMP one week ago, presents with a one-week history of severe lower abdominal
pain. Pain is constant, bilateral and accompanied by fever and chills. She has had some nausea and several episodes of
vomiting. She has been sexually active for 3 years and has had unprotected intercourse with several partners. She denies
irregular bleeding, dysmenorrhea or dyspareunia. Past medical history is non contributory. Past surgical history is re-
markable for tonsillectomy as a child and an uncomplicated vaginal delivery one year ago.

Physical exam reveals an ill appearing 16-year-old with a temperature of 98.6° F (37° Celsius) and has a pulse of 94 bpm,
BP 124/82 and a respiratory rate 22 breaths/minute. On examination of the abdomen, bowel sounds are present, there
is bilateral lower abdominal tenderness and the abdomen is slightly distended with rebound, negative psoas and Mur-
phy’s signs. Pelvic exam reveals the BUS (Bartholins, Urethral, Skene’s glands) to be normal and the vagina to be pink
TH
APGO MEDICAL STUDENT EDUCATIONAL OBJECTIVES, 10 EDITION
TEACHING CASES

and moist. There is a purulent discharge from the cervical os and the cervix appears indurated. The uterus is in the mid-
line position and is soft and tender to palpation. There is bilateral adnexal fullness and moderate tenderness.

Laboratory evaluation includes positive GC, negative RPR and WBC 17.6 with a left shift. Urinalysis is remarkable for
few WBC’s, no bacteria, no leukocyte esterase, no nitrites, 3+ ketones and negative urine HCG.

COMPETENCY-BASED DISCUSSION & KEY TEACHING POINTS:


Competencies addressed:
• Patient Care
• Medical Knowledge
• Interpersonal and Communication Skills
• Systems-Based Practice

1. What is your differential diagnosis for acute abdominal pain in a sexually active female?

• Salpingitis, appendicitis, ruptured ovarian cyst, ovarian torsion, ectopic pregnancy, acute pelvic inflamma-
tory disease

2. What is the most likely diagnosis in this case?

• Acute pelvic inflammatory disease (PID)

3. What are the most likely organisms responsible for this condition?

Likely pathogens include N gonorrhoeae, C trachomatis, anaerobes, gram positive and negative bacteria and
streptococci. Because of the polymicrobial nature of pelvic inflammatory disease (PID), all treatment regi-
mens must provide broad-spectrum coverage.

4. What are the common presenting signs and symptoms for this condition?

• The most common presenting complaint of women with PID is lower abdominal pain.
• Associated symptoms include vaginal discharge, irregular bleeding, dysmenorrhea, dyspareunia, dysuria,
nausea, vomiting and fever.
• Pelvic pain, fever and vaginal discharge are the most common findings if PID is secondary to gonococcal
infection. Patients may be asymptomatic if chlamydia is the causative organism. Women who have gono-
coccal infection have evidence of more acute inflammation (peritoneal signs, fever, leukocytosis) than
those who have nongonococcal infection because of the endotoxin produced by N gonorrhoeae.
• The clinical criteria necessary for the diagnosis of PID include:
• Abdominal tenderness +/- rebound
• Adnexal tenderness
• Cervical motion tenderness
• Plus one or more of the following: Gram stain of endocervix positive for Gram negative intracellular
diplococci, temperature >38 degrees C, WBC>10,000, pus on culdocentesis or laparoscopy, pelvic ab-
scess on bimanual exam or ultrasound
• Most women with acute PID present during the first half of the menstrual cycle. Presentation later in the
cycle indicates an infection of longer duration and increases the likelihood of a tuboovarian abscess
(TOA).

Copyright © 2014 by Association of Professors of Gynecology and Obstetrics (APGO).


For permissions: apgoadmin@apgo.org!
TH
APGO MEDICAL STUDENT EDUCATIONAL OBJECTIVES, 10 EDITION
TEACHING CASES

• Atypical presentations of PID are common and complicate the differential diagnosis. For example, the
symptoms of Fitz-Hugh-Curtis syndrome (right upper quadrant pain caused by liver capsule inflamma-
tion) may mimic hepatitis or cholecystitis.

5. What is the definitive diagnostic tool for equivocal cases?

• Laparoscopic findings of inflamed, dilated fallopian tubes, with purulent discharge

6. What criteria will you use to determine inpatient vs. outpatient treatment of PID?

• The decision to hospitalize a patient should be based on provider clinical judgement. However, in the fol-
lowing situations patients should be hospitalized:
! Surgical emergencies (e.g., appendicitis) cannot be excluded
! Patient is pregnant
! Patient does not respond clinically to oral antimicrobial therapy
! Patient is unable to follow or tolerate an outpatient oral regimen
! Patient has severe illness, nausea and vomiting, or high fever
! Patient has a tubo-ovarian abscess

7. What is your management and follow-up plan?

• Treatment for both chlamydia and gonorrhea


• Different regimens are adequate
• Outpatient treatment: ceftriaxone 250 mg IM plus 14 days of doxycycline 100 mg po BID.
• Inpatient treatment: cefotetan 2 gms IV q 12 hours plus doxycycline 100 mg IV/po q 12 hours until >
24 hours of clinical improvement, then continue outpatient regimen for a total of 14 treatment days
• All patients who are managed as outpatients must be reevaluated within 72 hours of the initiation of anti-
biotics. If there is not significant improvement in symptoms, hospitalization for re-evaluation should oc-
cur.
• Sex partners of PID patients should be examined and treated if they had sexual contact with the patient in
the 60 days prior to the onset of symptoms.
• Partners should be treated empirically for N gonorrhoeae and C trachomatis, regardless of the apparent eti-
ology of the PID or pathogens isolated from the infected woman. Without treatment of infected partners,
risk of reinfection is high.
• Repeat cultures are recommended about 3-4 weeks after treatment of Chlamydia infection to confirm
treatment efficacy and rule out asymptomatic reinfection.

8. If this condition went untreated, what would be the possible sequelae?

• Pelvic adhesions, tubal occlusion, chronic pelvic pain, ectopic pregnancy, infertility

9. How would one rule out a diagnosis of UTI in this patient?

• The patient should be questioned about any history of urgency, frequency, dysuria, or nocturia.
• Pyuria and bacteriuria on a microscopic exam markedly increase the chances that a patient has a UTI. A
urine dipstick which is leukocyte esterase positive is also suggestive that a patient may have a UTI. Since
this patient only had pyuria, no bacteriuria and urine dipstick for leukocyte esterase and nitrites was nega-
tive, it is unlikely that she has a UTI.

Copyright © 2014 by Association of Professors of Gynecology and Obstetrics (APGO).


For permissions: apgoadmin@apgo.org!
TH
APGO MEDICAL STUDENT EDUCATIONAL OBJECTIVES, 10 EDITION
TEACHING CASES

10. What are some STI prevention strategies?

• Abstinence
• Male and female condoms
• Pre-exposure vaccination: Gardisil and Cervarixare HPV vaccines that can prevent transmission of some
of the most common strains of HPV. The CDC recommends the hepatitis B vaccine is for all unvaccinat-
ed, uninfected persons being evaluated for an STI. Partner treatment: when partners are treated, patients
have reduced risk for reinfection. Patients with STIs should be encouraged to notify their sex partners
and urge them to seek medical evaluation and treatment.

REFERENCES

Beckman CRB, et al. Obstetrics and Gynecology. 7th ed. Philadelphia: Lippincott, Williams & Wilkins, 2013.

Hacker NF, Moore JG, et al. Essentials of Obstetrics and Gynecology. 5th ed. Philadelphia: Saunders, 2010.

Centers for Disease Control and Prevention. Sexually Transmitted Disease Treatment Guideline 2006.
www.cdc.gov/std/treatment/

ACOG Practice Bulletin 91, Treatment of UTI in Nonpregnant Patients, October 2005.

Copyright © 2014 by Association of Professors of Gynecology and Obstetrics (APGO).


For permissions: apgoadmin@apgo.org!
TH
APGO MEDICAL STUDENT EDUCATIONAL OBJECTIVES, 10 EDITION
TEACHING CASES
!

U N I T 2: O B S T E T R I C S
SECTION B: ABNORMAL OBSTETRICS

Educational Topic 23:


Third Trimester Bleeding

Rationale: Bleeding in the third trimester requires prompt evaluation and management to reduce maternal and fetal
morbidity and mortality.

Intended Learning Outcomes:!!

A student should be able to:

• List the causes of third trimester bleeding


• Describe the initial evaluation of a patient with third trimester bleeding
• Differentiate the signs and symptoms of third trimester bleeding
• List the maternal and fetal complications of placental previa and abruption placenta
• Describe the initial evaluation and management plan for acute blood loss
• List the indications and potential complications of blood product transfusion

TEACHING CASE

CASE: A 25-year-old G2P1 woman at 32 weeks gestation is brought to labor and delivery by her husband. About an
hour before, she was watching television when she noted a sudden gush of bright red blood vaginally. The bleeding was
heavy and soaked through her clothes, and she has continued to bleed since then. She denies any cramps or abdominal
pain. She says that her last sexual intercourse was a week ago. A review of her prenatal chart finds nothing remarkable
other than a borderline high blood pressure from her first prenatal visit that has not required medication. There is no
mention of bleeding prior to this episode. She had an ultrasound to confirm pregnancy at 14 weeks, but none since.

Physical examination reveals an extremely pale woman whose blood pressure is 98/60, pulse 130, respirations 30, tem-
perature 99° F. Her abdomen is soft without guarding or rebound to palpation, and the uterus is nontender and firm,
but not rigid. Fundal height is 33cm. Fetal heart tones are in the 140s with good variability. The external monitor re-
veals uterine irritability, but no discrete contractions are seen. There is a steady stream of bright red blood coming from
her vagina.
TH
APGO MEDICAL STUDENT EDUCATIONAL OBJECTIVES, 10 EDITION
TEACHING CASES

COMPETENCY-BASED DISCUSSION & KEY TEACHING POINTS:


Competencies addressed:
• Patient Care
• Medical Knowledge
• Systems-Based Practice

1. What is your differential diagnosis for potential causes of bleeding for this patient?

• Placental abruption
• Placenta Previa
• Vasa Previa
• Genital lacerations/trauma (e.g. labial, vaginal or cervical)
• Foreign body
• Cervical/vaginal cancer
• Cervicitis
• Bloody show

2. What steps would you take to evaluate this patient?

The key teaching points for this question is for the student to discuss a systematic approach to patient evalua-
tion, which includes not only identifying the etiology of the bleeding, but also evaluation of both the maternal
and fetal status.

• Assess maternal hemodynamic status:


• Serial vital signs
• Hematologic studies to assess for acute anemia and DIC
• Confirm placental location
• Avoid digital cervical exam
• Sonographic evaluation of placental location
• Assess fetal status:
• Continuous external heart rate monitor or sonographic biophysical assessment
• Kleihauer-Betke test for maternal-fetal hemorrhage

3. What signs and symptoms would help you differentiate the potential causes of the bleeding?

The key-learning outcome for this question is for the student to list the primary clinical characteristics that
differentiate abruption and previa and discuss the epidemiology and risk factors for each.

• Placental abruption:
• Epidemiology:
! Separation of the placenta from the uterine wall prior to delivery of the fetus
! Occurs in 1 in 100 births
! Accounts for approximately 30% of cases of third trimester bleeding
! 25% recurrence risk in a subsequent pregnancy

Copyright © 2014 by Association of Professors of Gynecology and Obstetrics (APGO).


For permissions: apgoadmin@apgo.org!
TH
APGO MEDICAL STUDENT EDUCATIONAL OBJECTIVES, 10 EDITION
TEACHING CASES

• Risk factors:
! Hypertension (chronic or gestational)
! Cocaine use/smoking
! Abdominal trauma
! Sudden uterine decompression (as with rupture of membranes)
! Preterm premature rupture of membranes
• Clinical presentation:
! Frequent uterine contractions or hypertonicity
! Vaginal bleeding (sometimes catastrophic)
! Non-reassuring fetal heart rate tracing
! Hypofibrinogenemia supports the diagnosis
! Disseminated intravascular coagulation occurs in 10% to 20% of severe abruption
• Placenta previa:
• Epidemiology:
! Occurs when placental tissue covers the cervical os.
! Central or total placenta previa – placenta completely covers the os
! Partial placenta previa – placenta partially covers the os (os must be partially dilated)
! Marginal previa - the placental edge is adjacent to the os, but does not cover it
! Low-lying placenta - the placenta approaches the os, but is not at its edge.
! At 24 weeks, about 1 pregnancy in 20 will demonstrate ultrasound evidence of a placenta
previa
! At 40 weeks, the incidence decreases to 1 in 200
! Accounts for approximately 20% of cases of third trimester bleeding
• Risk factors:
! Prior cesarean delivery
! History of myomectomy
! Increasing number of uterine curettages
! Increased parity
! Multiple gestation
! Advanced maternal age
! Smoking
• Clinical presentation:
! Bleeding is usually painless and may occur after intercourse
! Patients may also present with contractions, thus ultrasonography is critical to differentiat-
ing from abruption
• Vasa previa:
• Epidemiology:
! Fetal vessels of a velamentous cord insertion cover the cervical os
! Incidence is less than 1% of all pregnancies
• Risk factors:
! Multiple gestations: up to 11% in twins and up to 95% in triplets
• Clinical presentation:
! The diagnosis is suggested by painless vaginal bleeding in the absence of evidence of placen-
ta previa or abruption.

Copyright © 2014 by Association of Professors of Gynecology and Obstetrics (APGO).


For permissions: apgoadmin@apgo.org!
TH
APGO MEDICAL STUDENT EDUCATIONAL OBJECTIVES, 10 EDITION
TEACHING CASES

rd
• Other causes: causes of 3 trimester bleeding such as cervicitis, cervical erosions, trauma, cervical can-
cer, foreign body or even bloody show can usually be differentiated on physical exam once the pre-
ceding etiologies are ruled out.

4. What steps would you take to manage the low blood pressure and tachycardia that the patient is displaying?

The key-learning outcome for this question is for the student to recognize the signs and symptoms suggestive
of evolving shock related to acute blood loss and discuss the tenets of management that should include:

• Ensure adequate airway and assess vitals


! Serial blood pressure, heart rate, and respirations
! Continuous oxygen saturation monitor
• Establish adequate IV access
! 2 large bore IVs or central venous line
• Monitor blood and coagulation profiles
! Serial CBC and platelet counts
! Serial prothrombin time, partial thromboplastin time, and fibrinogen
• Volume resuscitation
! Crystalloid
! Packed red blood cells
! Platelets, fresh frozen plasma and cryoprecipitate as indicated
• Monitor vitals and response to therapy:
! Serial blood pressure, heart rate, and respirations
! Continuous oxygen saturation monitor
! Continuous urine output assessment via indwelling Foley catheter
rd
• Management of the patient with significant 3 trimester hemorrhage, when the fetus is mature, is hemo-
dynamic stabilization and delivery
• Vaginal delivery is generally precluded in the setting of abruption with persistent hemodynamic instabil-
ity
• Cesarean delivery is required for all cases of previa and vasa previa

5. Under what circumstances would you consider blood product transfusion?

The key teaching points for this question is for the student to list the possible parameters for blood transfu-
sion, in addition to listing the common transfusion complications:

• Indications for blood product transfusion


! Acute blood loss of 30-40% blood volume
! Chronic blood loss with hemoglobin <6g/dL or <10 g/dL in patients with cardiovascular or
pulmonary problems
! Abnormalities of the coagulation system
! Fibrinogen < 150 mg/dL
! Prolongation of PTT
! Platelets < 20,000
! Platelets < 50,000 and cesarean delivery

Copyright © 2014 by Association of Professors of Gynecology and Obstetrics (APGO).


For permissions: apgoadmin@apgo.org!
TH
APGO MEDICAL STUDENT EDUCATIONAL OBJECTIVES, 10 EDITION
TEACHING CASES

• Blood products

Product (mL) Contents Uses and effects

Whole Blood All compo- Rarely used. Only in the setting of massive
nents bleeding
(1 unit = 500mL)

Packed RBC RBC only One unit increased hematocrit by 3 percent-


age points
(I unit = 350 mL)

Frozen plasma All clotting Use for deficiencies in multiple clotting fac-
factors, no tors. One unit of FFP increased fibrinogen by
(1 unit = 200–300 mL) platelets 7-10 mg/dL

Cryoprecipitate Fibrinogen, Ten bags of cryoprecipitate will raise plasma


factors VIII, fibrinogen by 70 mg/dL in a 70 kg recipient
(1 bag – 10-15 mL) XIII, vWF

Platelets Platelets Six units of whole blood-derived or one unit


of apheresis-derived platelets will raise the
(1 unit = 50mL) platelet count by approximately 30,000/µL

• Complications
! Febrile non-hemolytic and chill-rigor reactions
! Acute hemolytic reaction due to ABO incompatible transfusion
! Delayed hemolytic transfusion reaction
! Transfusion-related acute lung injury
! Allergic reactions to unknown blood components
! Volume overload
! Graft vs. Host Disease (GVHD)
! Infectious complications (HIV, Hep B, Hep C, etc)

REFERENCES

Beckman CRB, et al. Obstetrics and Gynecology. 7th ed. Philadelphia: Lippincott, Williams & Wilkins, 2013.

th
Hacker NF, Moore JG, et al. Essentials of Obstetrics and Gynecology. 5 ed. Philadelphia: Saunders, 2010.

Copyright © 2014 by Association of Professors of Gynecology and Obstetrics (APGO).


For permissions: apgoadmin@apgo.org!

Você também pode gostar